Archer 8

Pataasin ang iyong marka sa homework at exams ngayon gamit ang Quizwiz!

C Choice C is correct. Professionals who routinely wear latex gloves, such as housekeepers, nurses, or hairdressers, are at a higher risk of developing a latex allergy than other populations. Reoccurring exposure to latex is responsible for this increased risk. Choices A, B, and D are incorrect. These populations are not at an increased risk of developing a latex allergy.

A community health nurse is evaluating different populations for risks of developing a latex allergy. Which of the following groups is at the highest risk of developing a latex allergy? A. Kindergarteners B. The homeless C. Hospital housekeepers D. Individuals with a lowered immune system

B Choice B is correct. Some drugs are frequently abused or have a high potential for addiction. Drugs that cause dependency are restricted to use in situations of medical necessity if they are allowed at all. According to law, drugs that have a significant potential for abuse are placed into categories called schedules. Choices A, C, and D are incorrect. Addiction refers to the overwhelming feeling that drives someone to use a drug repeatedly, although it is not medically necessary. Psychological dependence occurs when an individual has few signs of physical discomfort when a drug is withheld. However, the individual feels an intense, compelling desire to continue the use of the drug. Withdrawal is a term used to describe physical signs of discomfort that an individual experiences when a drug is no longer available.

An altered physical condition caused by the nervous system adapting to repeated drug use is: A. Addiction B. Physical dependence C. Psychological dependence D. Withdrawal

A Choice A is correct. The nurse needs to assess the client's symptoms in terms of onset, frequency, and severity. Through this assessment, the nurse will be able to identify patterns and collaborate with the health care provider (HCP) in scheduling round-the-clock and as needed (PRN) emetic therapy, as antiemetics as often administered before chemotherapy and repeated based on the response and duration of chemotherapy-induced nausea and vomiting. The nursing assessment will assist in identifying the response (and duration of the response) to these medications. Therefore, the nursing priority is coordinating with the client and health care provider (HCP) to ensure adequate control of chemotherapy-induced nausea and vomiting. Choice B is incorrect. Cold temperatures may decrease any odor(s) from food(s) that may trigger nausea and/or vomiting, so heating may not be helpful. Conversely, the client may request certain foods be heated before consumption. Regardless, the heating of food is not the priority. Choice C is incorrect. Although promoting relaxation and distraction techniques are important, these interventions are not the priority. Choice D is incorrect. A bland diet may help decrease the client's symptoms, but limiting the client to a soft, bland diet is not the nurse's most important role at this point. Regardless of which drugs are prescribed to prevent or reduce chemotherapy-induced nausea and vomiting, these medications are most effective when used with an evidence-based approach on a scheduled basis for prevention and management and when given before nausea and vomiting begin. When clients receive dose-dense chemotherapy, the intensity of chemotherapy-induced nausea and vomiting also increases, and more aggressive antiemetic therapy is needed. Teach clients to continue the therapy even when chemotherapy-induced nausea and vomiting appear to be controlled. Clients should take an antiemetic at the first sign of nausea to prevent it from becoming uncontrollable.

An ovarian oncology client receiving the chemotherapy agent cisplatin for cancer begins experiencing nausea and vomiting. The nurse's most important role when caring for this client is which of the following? A. Evaluating the onset, frequency, and severity of the client's nausea and vomiting B. Ensuring the food is heated before the client eats C. Providing venues for relaxation or distraction D. Limiting the client's intake to a soft, bland diet

A Choice A is correct. The term "audible" refers to the loudness of something. Characteristics of speech to evaluate include rate, rhythm, loudness, fluency, quantity, articulation, content, and pattern. Choice B is incorrect. Articulation refers to the production and use of sounds. Choice C is incorrect. Fluency is a speech-language pathology term that means the smoothness or flow with which sounds, syllables, words, and phrases are joined together when babbling. Choice D is incorrect. Speech quality refers to the characteristic features of an individual's voice.

Documenting the statement, "Normal speech is audible", would be a normal finding for which speech quality? A. Loudness B. Articulation C. Fluency D. Quality

B Choice B is correct. This patient has bruises on her thigh, wrists, and upper back that are in different stages of healing, which is a reliable indicator of abuse. Bruising on these parts of the body is not a common area for 2-year olds to injure. Typical areas of injury are the knees elbows and chin. Thigh, wrist, and upper back injuries can be due to grabbing, burning, or pushing. The department of child and family services (DCFS) needs to be contacted by the nurse because nurses are mandated reporters. Recognizing signs of abuse is extremely important. Choice A is incorrect. The parents do not need to be questioned at this point. Once the nurse calls DCFS, they will handle the questioning and investigation. Assessing these bruises is enough assessment findings to call DCFS. Choice C is incorrect. There is no indication to call poison control at this time. Choice D is incorrect. If the patient's respiratory status were unstable, this would be necessary. However, the patient is stable at this time on a bronchodilator and steroids.

Parents bring their 2-year old daughter into the emergency department after picking her up from her aunt's house. They are concerned that she has an upper respiratory infection. The nurse notices bruises on the patient's posterior thigh, wrists, and upper back. They appear to be in different stages of healing. Once the patient is stable after administering a bronchodilator and steroid, what should the nurse do? A. Question the parents B. Call the department of child and family services (DCFS) C. Call poison control D. Obtain an arterial blood gas (ABG)

B, D Choices B and D are correct. Impaired tactile sensation is often caused by peripheral neuropathy secondary to diabetes. Peripheral neuropathy, a long-term complication of diabetes, is characterized by the person's inability to feel things like heat, cold, and a painful stimulus like the prick of a needle in their feet. Impaired stereognosis is the lack of the client's ability to identify an everyday object with tactile sensations and without visual cues. Impaired stereognosis is associated with Alzheimer's disease. Choice A is incorrect. Grady Scales are used to determine levels of consciousness and not gustatory sensation. The impaired gustatory sensation is assessed by providing the client with small tastes of sweet, sour, salty, and spicy foods to identify their feelings. Choice C is incorrect. The impaired auditory sensation is assessed using an audiometer or a tuning fork. The Braden Scale is used to screen clients for their risk of developing a pressure ulcer. The Braden Scale uses scores from less than or equal to 9 to as high as 23. The lower the number, the higher the risk of developing a pressure ulcer. Score categories include 19-23 = no risk; 15-18 = mild risk; 13-14 = moderate risk or less than 9 = severe risk Choice E is incorrect. Proprioception is the sense of the relative position of one body segment to other body segments. Examples of tests to assess proprioception include the Finger-Nose test, the Heel-shin test, and the Thumb finding test. Morse scale assesses a patient's risk of falling, not proprioception. It consists of six variables that are quick and easy to score: History of falling - immediate or within three months; Secondary diagnosis; Ambulatory aids; Intravenous therapy; Gait and Mental status.

Select the sensory impairment that is accurately paired with one of its possible causes or a method for assessing it. Select all that apply: A. Impaired gustatory sensation: Using the Grady Scale B. Impaired tactile sensation: Diabetes C. Impaired auditory sensation: Using the Braden Scale D. Impaired Stereognosia: Alzheimer's disease E. Impaired Proprioception: Morse Scale

D Choice D is correct. The nurse should assess the patient first and provide any appropriate interventions to address the client's situation. Once the client's needs are met, other tasks can be performed. Choice A is incorrect. Alarms should not be turned off. A signal indicating decreased blood pressure requires immediate assessment. Choice B is incorrect. Blood pressure alarm limits should not be changed unless ordered by the physician. Choice C is incorrect. The patient should be assessed immediately as the patient's well-being holds priority over the convenience of the nurse.

The client in the intensive care unit is admitted because of a subdural hematoma. The nurses are currently in the middle of a clinical hand-off when suddenly the alarm goes off, indicating a drop in the client's blood pressure. The initial action of the nurses would be: A. Turn the alarm off and inform the incoming nurse of the decrease in blood pressure. B. Lower the blood pressure alarm limits on the monitor during the hand-off. C. Continue with the hand-off and include the drop in blood pressure in the report. D. Assess the client and then resume the hand-off.

C Choice C is correct. Checking for any duplication in medication should be the first action of the nurse to eliminate the risk of adverse effects on the client. Choice A is incorrect. Checking for drug interactions should be done after determining if there is any duplication of medications. Choice B is incorrect. The identification of side effects from medications can be made after the duplication of drugs is determined. Choice D is incorrect. Asking about family members helping with his medications is irrelevant to the problem of polypharmacy as of the moment.

The home health nurse is visiting an elderly client for the first time in his home. Upon assessment of the client, the nurse notices that the client has been taking 12 prescription medications and five over the counter medications. What is the nurse's most appropriate action? A. Check for drug interactions. B. Check for side effects from the medications. C. Check for any medication duplication. D. Ask the client if there are family members helping him with his medications.

2 mL

The primary healthcare provider (PHCP) prescribes 100 mg of amoxicillin oral suspension by mouth, four times a day. The medication label reads amoxicillin 250 mg per 5 mL. The nurse prepares to administer how many milliliters per dose? Fill in the blank.

B Choice B is correct. The most common area injured during lifting is the lumbar spine. This is because it supports the lower back. Choices A, C, and D are incorrect. These options are incorrect as the lumbar spine is most often the target of injuries while lifting. ADDITIONAL INFO Effective Measures to Prevent Back Injury Include Have the necessary assistance to move the object. Planning the move and communicating with the other individual who will assist you. Using the shoulder, upper arms, hips, and thighs as the predominant muscles to help with the move. Keep objects close to your body when lifting or carrying objects. Avoid twisting by using your feet to turn your body. Use a mechanical lift when necessary.

The occupational health nurse is conducting an in-service on reducing back injuries. It would be correct for the nurse to identify the most common location of the injury is the A. cervical spine. B. lumbar spine. C. thoracic spine. D. pelvis.

Check the pH of the eye Irrigate the eye from the inner canthus to the outer canthus Assess the visual acuity Document the occurrence Call the child's parent

The school nurse is attending to a student who got a chemical cleaner in her eyes. In which order should the following actions be performed? Irrigate the eye from the inner canthus to the outer canthus Check the pH of the eye Call the child's parent Assess the visual acuity Document the occurrence

B Choice B is correct. To monitor for signs of nephrotoxicity; the nurse should monitor the results of kidney function tests closely while the patient is taking gentamicin. Gentamicin is an aminoglycoside drug, that is capable of causing severe adverse effects in some patients. The most significant concerns are their effects on the inner ear and the kidneys. Damage to the inner ear, or ototoxicity, may cause hearing impairment, dizziness, persistent headache, or ringing in the ears. Nephrotoxicity is recognized by abnormal kidney function tests, such as elevated serum creatinine or blood urea nitrogen. Choice A is incorrect. Although intake and output (I&O) may be ordered, it is not indicated simply because of the use of gentamicin. Choice C is incorrect. Visual acuity tests are not required when a patient is taking gentamicin therapy. Choice D is incorrect. The primary concerns related to gentamicin therapy are ototoxicity and nephrotoxicity. Blood glucose monitoring is not indicated because of the use of this therapy.

There is a new patient in your clinic. Six months ago, he had a kidney transplant and is taking immunosuppressive drugs. Recently, he has been experiencing repeated bacterial infections and was switched to different antibiotics throughout the past six months. The physician suspected kidney infection. He is admitted to the hospital and administered gentamicin 300 mg daily by IV infusion. Which of the following tests should the nurse monitor? A. Input and output ratio B. Kidney function tests C. Visual acuity tests D. Fasting blood glucose levels

C Choice C is correct. When documenting an incident, you do not mention the occurrence report in the patient's medical record. In the chart, you would provide an objective description of what happened citing your observations, and then describe the follow-up actions taken. Choices A, B, and D are incorrect. These are components of the incident report that should be included. Therefore, these are incorrect answers to the question being asked.

Upon entering a patient's room, the nurse finds the patient lying on the floor of the bathroom. When documenting the incident on the patient's medical record, the nurse should include all of the following information, except? A. Provide an objective description of what happened. B. Report what the nurse observed. C. Note that an occurrence report was completed. D. Describe follow-up actions taken.

D Choice D is correct. A nonstress test (NST) is a non-invasive test performed in pregnancies over 28 weeks gestation. During the procedure, fetal heart rate and uterine contractions are recorded using external electronic monitors and correlated with fetal movements as reported by the mother. This test determines the fetus's condition during the third trimester of pregnancy. Choice A is incorrect. Cervical assessment estimates whether the cervix is favorable for labor induction. More specifically, the Bishop scoring system assesses cervical readiness for labor via five factors: cervical dilation, effacement, consistency, position, and fetal station. Choice B is incorrect. Blood sugar control can be assessed during pregnancy by random blood glucose assessments or, for a larger assessment, by glycated hemoglobin (HbA1c). Additionally, during pregnancy, several tests are used to identify gestational diabetes. The first, called the Glucose Challenge Screening, is a preliminary screening test performed between 26 and 28 weeks. If a woman tests positive during this screening test, the second test, called the Oral Glucose Tolerance Test (OGTT), may be performed. Choice C is incorrect. A nonstress test does not determine fetal age. Ultrasound measurement of the embryo or fetus in the first trimester is the most accurate method to establish or confirm gestational age.

A 38-week pregnant client is scheduled to undergo a nonstress test (NST). While speaking with the nurse, the client inquires regarding the purpose of this type of testing. The most appropriate response by the nurse would be which of the following? A. "This test determines whether you are ready for labor induction." B. "A nonstress test assesses your blood sugar control." C. "This testing provides an accurate determination of fetal age." D. "A nonstress test assesses the fetal condition in the third trimester."

D Choice D is correct. Coffee contains caffeine that is a stimulant. It causes vasoconstriction and increased blood pressure. Thus, it should be avoided by clients who have dysrhythmias. Choice A is incorrect. There is no evidence that hot beverages are not suitable for the heart. Choice B is incorrect. Even if caffeine is ordered by the doctor to be avoided, the nurse should provide a much more comprehensive explanation to the patient as to why he should not drink coffee. Choice C is incorrect. Tea also contains caffeine and should also be avoided by the client.

A 45-year-old man is admitted to the telemetry unit for observation. After waking up in the morning, the client asks the nurse if there is any coffee. What should be the nurse's best response to the client? A. "Hot beverages are not allowed because of the condition of your heart." B. "Coffee was not in your diet order by the physician" C. "We don't have coffee in the unit. I can bring you some tea if you like." D. "As of the moment you cannot have coffee because it has caffeine, and caffeine can affect your heart."

D Choice D is correct. Children with ADHD exhibit short attention spans due to a variety of factors. Inattention tends to appear when a child is involved in tasks that require vigilance, rapid reaction time, visual and perceptual search, and systematic and sustained listening. Choice A is incorrect. Hyperactivity involves excessive motor activity. Children, particularly younger ones, may have trouble sitting quietly in environments where they are expected to do so (e.g., in school or church). Therefore, a child with ADHD would exhibit increased psychomotor activity, not lethargy. Choice B is incorrect. A child with ADHD is distracted by external factors and is very much distracted by external environmental stimuli. Choice C is incorrect. Poor verbal skills are not associated with ADHD. ADHD involves inattention, hyperactivity/impulsivity, or a combination and typically appears before the age of twelve. Diagnosis is made using clinical criteria. Approximately 20 to 60% of children with ADHD have learning disabilities, although most school dysfunction occurring in children with ADHD is due to inattention (resulting in missed details) and impulsivity (resulting in responding without thinking through the question). Treatment usually includes pharmaceutical therapy with stimulant medications, behavioral therapy, and educational interventions. Manifestations tend to diminish with age, but adolescents and adults may experience residual difficulties.

A 5-year-old child suspected of having attention-deficit/hyperactivity disorder (ADHD) is brought to the pediatric clinic for an evaluation. Which behavior, if observed by the clinic nurse, would support the suspicion of ADHD? A. Lethargy B. Preoccupation with body parts C. Poor verbal skills D. Short attention span

A Choice A is correct. Primary prophylaxis to prevent the initial variceal bleeding episode is one of the most important strategies for reducing mortality in cirrhotic clients. As such, client education plays a significant role in managing esophageal varices. Lifting heavy objects, straining during defecation, stretching, and the Valsalva maneuver may cause a marked increase in variceal pressure and should, therefore, be avoided by clients with esophageal varices, cirrhotic clients, and those with portal hypertension. Choice B is incorrect. Brisk walking is not contraindicated in clients with esophageal varices. Choice C is incorrect. The use of barbiturates is not contraindicated in clients with esophageal varices. Choice D is incorrect. The use of antacids is not contraindicated in clients with esophageal varices. Variceal bleeding is a consequence of portal hypertension, which, in turn, is the major complication of liver cirrhosis. The risk of variceal bleeding increases as variceal pressure increases. Variceal rupture is the most common fatal complication of cirrhosis. The severity of liver disease correlates with the presence of varices and the associated risk of bleeding. Bleeding from esophageal varices is the third most common cause of upper gastrointestinal bleeding (following only duodenal and gastric ulcers).

A 52-year-old client with a 20-year history of alcohol abuse is hospitalized with mild ascites, jaundice, and bruising. Imaging demonstrates the presence of esophageal varices, while the client's elevated serum ammonia level indicates hepatic encephalopathy. The nurse is concerned the client's esophageal varices may rupture and proceeds to educate the client accordingly. Which item should the nurse include in the client's education session? A. "Do not lift heavy objects." B. "Avoid walking briskly." C. "Avoid taking barbiturates." D. "Avoid ingesting antacids."

C Choice C is correct. Platelets play vital roles in hemostasis and thrombosis and can be inhibited by nonsteroidal anti‐inflammatory drugs (NSAIDs). NSAIDs reduce platelet adhesiveness, therefore impairing coagulation. Choice A is incorrect. NSAIDs do not impair the body's ability to synthesize vitamin K. Choice B is incorrect. The conversion of prothrombin (into thrombin) is not inhibited by the use of NSAIDs. Choice D is incorrect. NSAID use does not result in the destruction of factor VIII. Typically, NSAIDs possess analgesic, anti-inflammatory, and antiplatelet effects. NSAIDs should be used cautiously in clients with renal insufficiency. In clients in which long-term NSAID therapy is anticipated, it is prudent to monitor the client for occult blood in stool and changes in the complete blood count (CBC), electrolytes, and hepatic and renal function. Acetaminophen is a unique NSAID, possessing no anti-inflammatory or antiplatelet effects. Additionally, acetaminophen does not cause gastric irritation. Aspirin is the least expensive NSAID. This medication also has irreversible antiplatelet effects and increases the risk of gastrointestinal bleeding.

A 55-year-old client with osteoarthritis develops coagulopathy due to long-term NSAID use. The nurse caring for the client understands that the client's coagulopathy is mainly the result of: A. Impaired synthesis of vitamin K B. Blocked prothrombin conversion C. Decreased platelet adhesiveness D. Destruction of factor VIII

D Choice D is correct. Abused children have difficulty in social situations with peers. The effect of abuse on children becomes more apparent when the child reaches school age. At this point, difficulties develop in forming relationships with both peers and teachers. Choice A is incorrect. A victim of child abuse is likely to display inappropriate behavior in every (or nearly every) situation due to delayed development of social skills, insecurity, anxiety, or distrust. Choice B is incorrect. An abused child will often appear fearful and withdrawn from one or both caregivers involved in abusing the child. Choice C is incorrect. Abuse and neglect are often associated with physical injuries, delayed growth and development, and mental problems. These issues often result in lackluster performances at school. Generally, abuse can be attributed to a breakdown of impulse control in the parent or caregiver. The parent's childhood may have lacked affection and warmth, may not have been conducive to the development of adequate self-esteem or emotional maturity, and, in many cases, also included other forms of maltreatment or abuse. Drug or alcohol use may provoke impulsive and uncontrolled behaviors toward their children. Parental mental disorders also increase the risk of abuse.

A 6-year-old is admitted to the hospital for multiple fractures and is subsequently identified as a victim of child abuse. A community health nurse (CHN) is assigned to the child following discharge from the hospital. Which characteristics would the CHN expect to see from the child? A. The child displays appropriate behaviors during each encounter. B. The child seeks to contact his or her parents. C. The child becomes an overachiever in school and displays advanced developmental growth. D. The child becomes aloof and isolated from peers.

B Choice B is correct. Metformin, a medication used to treat type 2 diabetes, should be held 24 hours before a procedure that uses iodine dye to reduce the risk of lactic acidosis. The drug may be resumed about 48 hours after the procedure. Choice A is incorrect. Labetalol is used to treat hypertension and is safe to take before and after an iodine-based computed tomography scan. Choice C is incorrect. Levodopa, a medication used to treat Parkinson's disease, is safe to take before and after an iodine-based computed tomography scan. Choice D is incorrect. Ondansetron, a medication used to treat nausea and vomiting, is safe to take before and after an iodine-based computed tomography scan. The most common side-effect associated with Metformin initiation is gastrointestinal side effects such as bloating, diarrhea, nausea, or vomiting. The nurse should counsel the client that these side effects are transient and may be lessened by taking the medication with food. Metformin should not be taken within 48 hours of a contrast procedure because should contrast-related nephrotoxicity occur, Metformin metabolites accumulate and cause lactic acidosis.

A client is scheduled to undergo a computed tomography scan with iodine-based contrast dye. Which of the following medications may cause interaction and should be withheld for 24 hours before the procedure? A. Labetolol B. Metformin C. Levodopa D. Ondansetron

B Choice B is correct. Asking the client to elaborate on their concern is the most logical and therapeutic action. The client is likely misinformed about the disease transmission of HIV, and the nurse should encourage the client to verbalize their concerns. It also is appropriate for the nurse to respond to any misconceptions the client may have with compassion and facts. Choices A, C, and D are incorrect. These actions are incorrect. Based on the information provided, the client should not be relocated to a private room. Additionally, if the client were to be relocated, a private room would not be necessary, as another semi-Private room would be appropriate. The risk manager does not need to be notified of the request. This request would be irrelevant to a risk manager. Finally, an additional divider is unnecessary because this would further fuel the hysteria surrounding HIV. A client with HIV can be roomed with another individual, and standard precautions should be utilized. The worst symptom of HIV is the stigma. Many misconceptions exist regarding the transmission of HIV.

A client requests to change rooms after overhearing that their roommate is positive for the human immunodeficiency virus (HIV). The nurse should take which appropriate action? A. Relocate the client to a private room B. Ask the client to elaborate on their concern C. Notify the risk manager of the request D. Place an additional divider in-between the two beds

B Choice B is correct. Hypothermia is defined as a core body temperature of less than 95° F ( 35°C). Hypothermia is staged into Mild, moderate, severe, and profound hypothermia ( stages I to IV). Staging helps guide the treatment recommendations. A core body temperature between 90 to 95° F (32.2°C to 35°C) is considered mild hypothermia whereas a temperature between 82° F to 90° F (27.8°C to 32.2°C) is considered moderate. A core temperature less than 82° F (27.8°C) is severe. However, measuring the core body temperature accurately is challenging. Therefore, a model based on the vital signs and clinical symptoms called the "swiss staging model" is used to stage hypothermic patients. Based on the clinical manifestations as well as the stated core body temperature of 88° F given in this question, this client has moderate hypothermia ( Stage II). Manifestations of moderate hypothermia include decreased level of consciousness (LOC), hypoventilation, bradycardia, atrial fibrillation, hypovolemia, cessation of shivering, and possible hyperglycemia. Choice A is incorrect. Mild hypothermia (Stage I) presents with shivering, bradycardia, or tachycardia. The patient may also be alert or slightly confused. Choice C is incorrect. Severe hypothermia (Stage III) manifests as coma, fixed and dilated pupils, bradycardia, apnea, hypotension, ventricular fibrillation, and/or asystole. The core body temperature is typically less than 82° F Choice D is incorrect. Frostbite is hypothermia in the extremities. Frostbitten areas may appear red and swollen or may be pale in color. Blisters containing clear or bloody dark fluid may appear.

A man is found lying on the ground, confused, covered with snow, and is brought to the emergency department to be treated. The nurse checks the client's temperature and notes that it is 88° F (31.1 °C). The nurse also notes respirations of 10, a pulse of 50, and blood pressure of 79/52 mm Hg. The nurse correctly identifies that the client is experiencing which of the following conditions? A. Mild hypothermia B. Moderate hypothermia C. Severe hypothermia D. Frostbite

A Choice A is correct. Although this pediatric client initially presented with a severe asthma attack which has since resolved, this night shift nurse is correct in providing instruction regarding nocturnal sleep positioning, as nocturnal asthma exacerbation is a concern for this pediatric client. Nocturnal asthma symptoms are common and more pronounced in clients with more severe forms of asthma. This client, having required hospitalization due to a severe asthma attack, should be considered to have a more severe form of asthma. Here, the nurse should instruct the client to sleep in a high or semi-Fowler's position, as these two positions have been shown to be the most effective in preventing nocturnal asthma exacerbations. Choice B is incorrect. Sleeping in a prone position has been shown to increase the likelihood of a nocturnal asthma exacerbation. Choice C is incorrect. Sleeping in a side-lying position has been shown to increase pulmonary constriction and, subsequently, the likelihood of nocturnal asthma exacerbations. Choice D is incorrect. Sleeping in a supine position has been shown to increase the likelihood of nocturnal asthma exacerbations, primarily due to the worsening of several known contributing factors (i.e., gastroesophageal reflux disease and/or obstructive sleep apnea) and should therefore be avoided. In those with asthma, clients with more severe disease are significantly more likely to experience nocturnal asthma. The chances of experiencing asthma symptoms or an exacerbation are higher during sleep. Posture is an important determinant of pulmonary mechanics, which has critical implications for clients with asthma. Although the exact causation of nocturnal asthma has yet to be identified, certain contributing factors have been identified. Many of these contributing factors are exacerbated by the choice of one's sleeping position, including, but not limited to: Gastroesophageal reflux disease (GERD) Obstructive sleep apnea (OSA) Sinusitis (more specifically, postnasal drip)

A night shift nurse is caring for a pediatric client admitted earlier in the day following a severe asthma attack. To promote comfort at the time the client is going to bed for the night, the nurse instructs the client to assume which position? A. High or semi-Fowler's position B. Prone position C. Side-lying position D. Supine position

D Choice D is correct. Sexual pleasure is heightened during the second trimester of pregnancy. In the second trimester, most women experience significant relief from the discomforts of early pregnancy (nausea and vomiting, breast tenderness). The uterus is not too large to interfere with comfort and rest. The second trimester is also the time when pelvic organs are congested with blood, increasing pleasure in sexual activities. Choices A and B are incorrect. As long as risk factors such as preterm labor or incompetent cervix are not present, intercourse should not harm the pregnancy. Sexual intercourse should not be a cause of concern even in the third trimester unless risk factors such as preterm labor or placenta previa are present. Choice C is incorrect. Many women experience changes in sexual desire at different stages in pregnancy, depending on their general sense of well-being and the presence of certain discomforts brought about by the pregnancy. It is not the same throughout pregnancy.

A nurse at an obstetric clinic has conducted a teaching class on sexuality during pregnancy. Which of the following comments from a participant would indicate that the teaching has been effective? A. "At around the time I would normally have my period, I should abstain from intercourse." B. "I should no longer have sex during the last trimester of pregnancy." C. "My sexual desire will remain the same for the entire pregnancy." D. "The best time to enjoy sex is in the second trimester."

C Choice C is correct. Drooling in epiglottitis means that the child is having difficulty swallowing. This increases the risk of airway compromise. This patient should be seen by a physician immediately, and an emergency bedside tracheostomy prepared. Choice A is incorrect. A child with a urinary tract infection is expected to display dysuria and fever. This client should not be a priority over a patient in an emergency. Choice B is incorrect. The child diagnosed with leukemia is expected to have petechiae. This client should not be a priority over a patient in an emergency. Choice D is incorrect. A child with otitis media is expected to have a fever. This client should not be a priority over a patient in an emergency.

A nurse in the emergency department of a children's hospital is triaging patients. Which patient should the nurse arrange for the doctor to see first? A. A febrile 8-year-old girl complaining of pain during urination. B. A child diagnosed with leukemia displaying petechiae. C. A child diagnosed with acute epiglottitis two days ago and is drooling. D. A child with otitis media having fever.

B, D, E, F Choices B, D, E, and F are correct. Choice B is correct. The nurse should always approach the client from the unaffected side. Here, the nurse would approach this client from the client's left side to avoid startling the client. Choice D is correct. Activities that increase intraocular pressure, such as bending down, should be avoided. Choice E is correct. In order to prevent unwarranted injury, the client should always be oriented to his or her environment. Choice F is correct. A prescription for a stool softener is provided for multiple reasons. First, activities that increase intraocular pressure should be avoided. Since constipation and straining during defecation often increase intraocular pressure, stool softeners are administered to prevent constipation prophylactically. Second, any use of opioid pain medication during the surgical or postoperative procedure would likely inhibit gastrointestinal and colonic motility. A stool softener would assist in alleviating this medication side effect. Choice A is incorrect. During the postoperative period, activities that increase intraocular pressure should be avoided. Increased intraocular pressure can lead to postoperative vision loss and recurrent retinal detachment. When a client is placed in a prone position, the client's intraocular pressure increases significantly due to the position change. If the client remains in the prone position, the client's intraocular pressure will continue to rise over the next ten minutes while the client remains in this position. To avoid this increase in intraocular pressure, the client should lie down on their back (supine) or on the unaffected side (i.e., here, the client's left side would be the client's unaffected side) to reduce the intraocular pressure in the affected eye. Choice C is incorrect. As discussed above, activities that increase intraocular pressure should be avoided during the postoperative period, as increased intraocular pressure can lead to postoperative vision loss and recurrent retinal detachment. Clients should not be instructed to perform deep breathing and coughing exercises, as coughing causes increased intraocular pressure and should therefore be avoided. Intraocular pressure (IOP) occurs when a measurement is greater than 21 mm Hg. Retinal detachment is painless. Clients experiencing retinal detachment usually see an increase in floaters (i.e., objects that appear to move through a client's field of vision) or many flashes of bright light that last less than a second (photopsia) and have blurred vision. Peripheral vision is typically lost first, and vision loss spreads as the detachment progresses, causing grayness in the field of vision or resembling a curtain or veil falling across the line of sight. Most retinal detachments can be repaired via surgery, although the type of surgery is dependent upon the severity of the retinal tear.

A nurse is caring for a client following the surgical repair of a detached retina in the client's right eye. Which nursing action(s) should the nurse include in the client's plan of care? Select all that apply. A. Position the client in a prone position B. Approach the client from the left side C. Instruct the client to perform deep breathing and coughing exercises D. Instruct client to avoid bending down E. Orientate the client to the environment F. Obtain a prescription for a stool softener

A Choice A is correct. Checking perineal pads every shift is an incorrect practice and therefore the correct answer to this question. The nurse should assess the perineal pad of the immediate post-partum woman every 30-minutes, not every turn. Perineal pads getting soaked with blood within 30 minutes should be a cause of concern for the nurse for this is a sign of continuous bleeding through the uterus due to uterine atony. Choice B is incorrect. Palpating the fundus frequently is correct practice. The nurse should palpate the patient's fundus frequently to make sure that it is firm and contracted. A firm and contracted uterus prevents blood loss. Choice C is incorrect. Weighing used perineal pads once they are changed is correct practice. The nurse should weigh the perineal pads after they are soaked to accurately assess the amount of blood lost by the patient through the perineum. One gram in weight is equivalent to 1 mL in plasma. Taking note of the time that the pads were changed would signify the frequency of pad changes, which is also essential in the assessment. Choice D is incorrect. Checking vital signs for signs of shock is correct nursing practice. The nurse should assess the patient frequently for signs of trauma. These include low blood pressure, weak, thready pulses, increased heart rate, and increased respiratory rate.

A nurse is caring for a woman that just had a normal delivery an hour ago. The nurse understands that the patient is still at risk for uterine atony at this stage. All of the following interventions should be included in the care plan of the patient for detection of uterine atony, except: A. Checking for saturated perineal pads every shift B. Palpating the fundus at frequent intervals C. Weighing perineal pads once they are changed, noting the time it was changed and the saturation D. Checking vital signs frequently for signs of shock

C Choice C is correct. Clients with fecal impaction often pass watery mucus or fecal material around the impacted mass, mimicking diarrhea (also referred to as overflow diarrhea or paradoxical diarrhea). Abdominal cramping may or may not occur when the client passes this liquid stool. Due to the hardness of the stool present at the site of the client's fecal impaction, the bowel will often begin to leak out watery stool around the hardened stool. These watery stools will leak out of the rectum and present like watery diarrhea. Choice A is incorrect. An acute perforated ulcer is a frequent cause and presents with characteristic "board-like" rigidity of the abdominal wall, not fecal impaction. Choice B is incorrect. A common symptom of clients experiencing fecal impactions is the intense need to have a bowel movement. Choice D is incorrect. In clients with fecal impaction, the client's abdomen typically appears distended and swollen. Upon palpation, the client's stomach is hard to the touch. A fecal impaction is a mass of dry, hard stool that cannot be eliminated by a normal bowel movement. Fecal impaction, which may cause or develop from constipation, is common among older clients, particularly with prolonged bed rest or decreased physical activity. Prompt identification and treatment minimizes the risks of complications such as bowel obstruction leading to aspiration, stercoral ulcers, perforation, and peritonitis. Fecal impactions are treated initially with enemas of tap water followed by small enemas (100 mL) of commercially prepared hypertonic solutions. If unsuccessful, manual fragmentation and disimpaction of the mass may be necessary. Manual fragmentation and disimpaction of the mass is painful, so perirectal and intrarectal application of local anesthetics (e.g., lidocaine 5% ointment or dibucaine 1% ointment) is recommended. Some clients may require sedation. Recurrence is common and can be managed by increasing dietary fiber content to 30 gm/day, increasing water intake, and discontinuing medications that can contribute to colonic hypomotility.

A nurse is evaluating an 83-year-old client hospitalized after a fall. The client has not had a bowel movement for five days, and a fecal impaction is suspected. Which assessment finding would be most indicative of fecal impaction? A. Rigid, board-like abdomen B. The client has lost the urge to defecate C. Liquid stools D. Complaints of abdominal pain without distention

B Choice B is correct. Patent vascular access of at least a 20-gauge catheter is necessary before the infusion of intravenous contrast. Extravasation of contrast media can be severe, and treatment involves stopping the infusion, removing the catheter, and elevating the extremity above the heart. This can be avoided by establishing IV patency before the infusion of contrast. Warm or cold compresses may also be helpful. Choice A is incorrect. Shellfish allergies are to tropomyosin (a muscle protein), not iodine. Thus, asking questions regarding shellfish allergy before giving an intravenous contrast agent is unnecessary and has been disproven by the American College of Radiology for quite some time. The cause of contrast reactions is unclear, and previous reactions specific to iodine contrast should be inquired upon before the exam. Choices C and D are incorrect. Capillary blood glucose should be obtained before a PET scan - not CT. The client should be instructed to increase their fluid intake - not decrease, as contrast may be nephrotoxic. Computed tomography scans may be performed with or without contrast. This procedure uses radiation and is commonly indicated for abdominal pain, stroke, and spinal cord injuries. If contrast is administered, a baseline creatinine and eGFR should be obtained. The client should be instructed to increase their fluid intake following a contrast procedure to facilitate its passing.

A nurse is preparing a client for computed tomography (CT) scan with intravenous (IV) iodinated contrast. The nurse should take which action? A. Ask the client if they are allergic to shellfish B. Insert a 20-gauge peripheral vascular access device C. Obtain capillary blood glucose (CBG) D. Instruct the client to decrease their fluids after the procedure

C Choice C is correct. During the initial post-burn period, fluid shifting occurs, causing large amounts of plasma fluid to extravasate into interstitial spaces. Hypovolemia, causing hypoperfusion of burned tissue and sometimes shock, can result from fluid losses due to burns that are deep or that involve large parts of the body surface; whole-body edema from the escape of intravascular volume into the interstitium and cells also develops. Also, insensible fluid losses can be significant. The best intervention to address the fluid loss is administering intravenous fluids as ordered. Choice A is incorrect. Dopamine, a vasoconstrictor, would raise the client's blood pressure but would not prevent hypovolemia in a post-burn client. Choice B is incorrect. Medical anti-shock trousers are applied when the client is experiencing hypovolemic shock, not as a preventative measure. Choice D is incorrect. Although fresh frozen plasma may assist in preventing hypovolemic shock, using fresh frozen plasma to do so is expensive, carries a risk of disease transmission, and offers no advantage over traditional intravenous fluids and is therefore not the best choice. The Parkland formula is a calculation used to estimate the amount of replacement fluid required for the first 24 hours in a burn client to ensure the client is hemodynamically stable. The Parkland and other burn fluid resuscitation formulas are only a starting point; fluid volume and rate are adjusted based on clinical response. The most common systemic complications are hypovolemia and infection. Hypoperfusion of burned tissue may also result from direct damage to blood vessels or vasoconstriction secondary to hypovolemia.

A nurse is taking care of a client with severe burns. Because of fluid shifting, the nurse knows that the focus of attention is preventing hypovolemic shock. Which is the best intervention to address this? A. Administer dopamine as ordered B. Apply medical anti-shock trousers C. Infuse IV fluids as indicated D. Infuse fresh frozen plasma

C Choice C is correct. Nurses are required to report colleagues who are suspected of substance abuse, as this jeopardizes the safety of clients and other staff members. The initial action is to report this individual to the nursing supervisor on duty, as this individual can intervene immediately, therefore circumventing harm to clients and other staff members. Following the initial response by the nursing supervisor on duty, the issue will be escalated within the healthcare facility before being relayed to the applicable state board of nursing, which possesses complete jurisdiction to order and supervise the treatment of the impaired nurse. Choice A is incorrect. Calling security and having the colleague restrained would be appropriate if a disturbance had or is anticipated to occur, but nothing in the above scenario indicates this has or is anticipated to occur. Therefore, calling security and having the colleague restrained is inappropriate and would only inflame the existing situation. Choice B is incorrect. Although the police may be contacted at some point in the future, this is not the initial action. Choice D is incorrect. In the absence of posing a significant threat to clients or other staff members, locking the colleague in the medication room "until help is at hand" is grossly inappropriate and potentially criminal behavior by this nurse.

A nurse on the night shift enters the medication room and inadvertently discovers a colleague with a tourniquet wrapped around their upper arm, ready to inject a clear liquid into their antecubital area. Which initial action should the nurse take? A. Call security and have the colleague restrained B. Call the police and file charges against the colleague C. Call the nursing supervisor D. Lock the colleague in the medication room until help is at hand

B Choice B is correct. An increase in temperature leads to increased metabolism and cardiac workload. Choice A is incorrect. Although there can be diaphoresis during an increase in temperature, it is not going to be a reason to call the physician. Choice C is incorrect. Fever is not an early sign of cerebral edema. Choice D is incorrect. When there is significant blood loss, there is no increase in temperature. Instead, there will be a decrease in temperature.

A post-coronary artery bypass graft patient developed a fever of 38.8° C. The nurse notifies the physician of the elevated temperature because: A. The elevated temperature may lead to profuse sweating. B. It may increase cardiac output. C. It is a sign of cerebral edema. D. It is indicative of hemorrhage.

B Choice B is correct. Combining condoms and spermicidal contraceptive foam is highly effective in preventing pregnancy. In addition to being easily accessible, this method is also relatively inexpensive (often free if received from a local public health department). This combination is the most appropriate contraceptive method for this client. Choice A is incorrect. A hormonal contraception method such as medroxyprogesterone acetate injections increases the risk of clotting and strokes in women who smoke ten or more cigarettes daily. Therefore, following a review of the client's medical record and risk factors, this specific contraception method is not the best option for this client. Choice C is incorrect. Natural family planning involves intricate planning and timing of sexual contact around the menstrual cycle and signs of ovulation. Though this can be an effective method of birth control, it entails motivation, maturity, and strict attention to detail. Choice D is incorrect. The use of oral contraceptives would not be the ideal choice for this client. The use of oral contraceptives is contraindicated in women who are < 21 days postpartum or women who smoke > 15 cigarettes/day. Therefore, oral contraceptives are not the ideal choice for this client. The incidence of deep venous thrombosis and thromboembolism (e.g., pulmonary embolism) increases as the estrogen dose increases. Pregnancy rates tend to increase during the first year of use and decrease in subsequent years as users become more familiar with the chosen contraceptive method. In the United States, 12% of women who use contraception use intrauterine devices (IUDs).

A postpartum client is preparing to be discharged home with her full-term newborn. Prior to discharge, the client verbalizes, "I really should not get pregnant again in the next three years so I can finish college." History reveals that she smokes a pack of cigarettes a day. Which method of contraception would be the most appropriate for this client? A. Medroxyprogesterone acetate injectable suspension B. Condoms and spermicidal contraceptive foam C. Natural family planning D. Oral contraceptives

A Choice A is correct. Pregnancy increases a woman's risk of developing gingivitis and cavities. The patient has gingivitis and it appears like her dentist recommended tooth extraction for her. There is some concern regarding undergoing dental extractions during pregnancy. However, there is no evidence that a pregnant woman will need to delay dental removal. Delaying dental care could be harmful to the mother and fetus. Emergency treatment can be done at any time during pregnancy. However, elective dental surgery should be deferred until the second trimester (weeks 14 through 20). By the second trimester, fetal organogenesis is complete, and the risk of adverse effects from procedure/medications is lower. The consequences of not treating a dental infection during pregnancy outweigh the possible risks of the medications. The patient has gingivitis that needs to be addressed soon. Also, the blood volume of the pregnant woman significantly increases in 3rd trimester and remains elevated until delivery. Although there is no risk to the fetus during this trimester, the pregnant woman may experience increased discomfort. There is also a risk of hypotension in the supine position, so, short dental appointments are advocated during the 3rd trimester. It is, therefore, better not to perform elective dental extractions during 3rd trimester. Choices B and C are incorrect. The prenatal client should not delay dental care. Choice D is incorrect. Anti-viral medications should not be given to a woman who will be undergoing a dental procedure.

A prenatal client with gingivitis at her fourth-month clinic visit mentions that she has a tooth extraction planned for the following month and is wondering whether or not she can continue with the procedure. What information will you provide the prenatal client? A. The second trimester is the safest period for dental extractions. B. She will need to wait until after delivery to have the procedure performed. C. She should wait until the third trimester to have the procedure performed. D. She should take anti-viral medications before the procedure to prevent illness.

D Choice D is correct. Culture is defined as the customs, arts, social institutions, and achievements of a particular nation, people, or another social group. Choice A is incorrect. Society is defined as the people who live in a country or region, their organizations, and their way of life. Choice B is incorrect. A community is defined as all the people living in an area or a group or groups of people who share common interests. Choice C is incorrect. Spirituality is defined as the quality of being concerned with the human spirit or soul as opposed to material or physical things.

A shared, learned, and symbolic system of values, beliefs, and attitudes that shape and influence the way people see and behave within the world is defined as: A. Society B. Community C. Spirituality D. Culture

B Choice B is correct. It is the nurse's responsibility to be the client's advocate. She should be responsible for finding someone that can help the client with her request. Choice A is incorrect. Living wills can be changed by the client anytime and as many times as they wish as long as they are competent in making decisions. Choice C is incorrect. Living wills can be changed by the client anytime and as many times as they wish as long as they are competent in making decisions. Choice D is incorrect. The client does not need to ask permission from her lawyer to change her living will.

After talking to her family, an elderly client says that she wants to change the living will she wrote two weeks ago. The nurse's most appropriate reply would be: A. "You can only change your living will a year after it is formulated." B. "Let me see if I can find someone to help you." C. "You can only make changes to your will after 3 weeks." D. "Let's call your lawyer first and see what he thinks."

D Choice D is correct. Performing and documenting a focused assessment on skin integrity is appropriate since this is a newly identified problem. Choices A and B are incorrect. The initial assessment stands as-is and cannot be redone or correct. Choice C is incorrect. This is not a life-threatening event. Therefore, there is no need for an emergency assessment.

Although you were informed that your assigned patient has no special skincare needs, upon your assessment, you observe reddened areas over bony prominences. What action is appropriate? A. Correct the initial assessment form B. Redo the assessment and document current findings C. Conduct and document an emergency assessment D. Perform and document a focused assessment of skin integrity

C Choice C is correct. Prolonged inadequate nutrition causes weight loss, muscle atrophy, and the loss of subcutaneous tissue. These three conditions reduce the amount of padding between the skin and bones, thus increasing the risk of pressure ulcer development. More specifically, inadequate intake of protein, carbohydrates, fluids, zinc, and vitamin C contributes to pressure ulcer formation. Several factors contribute to the formation of pressure ulcers: friction and shearing, immobility, inadequate nutrition, fecal and urinary incontinence, decreased mental status, diminished sensation, excessive body heat, advanced age, and the presence of certain chronic conditions. Choice A is incorrect. Incontinence can be a source of skin irritation, but it is not a stand-alone risk factor for developing a decubitus ulcer. Choice B is incorrect. While diabetics may be prone to developing skin ulcers and may experience slower healing times, an ambulatory patient is not at risk for developing pressure sores. Choice D is incorrect. Obesity is not a risk factor for developing a decubitus ulcer.

Among the patients in a long term care facility. Which client is at the highest risk for developing a decubitus ulcer? A. An incontinent client who had 3 diarrheal stools. B. An 80-year-old ambulatory diabetic client. C. A 79-year-old malnourished client on bed rest. D. An obese client who uses a wheelchair.

A Choice A is correct. Mineral oil is contraindicated in pregnancy as it decreases nutrient absorption in the mother. Choice B is incorrect. Sleeping in a side-lying position removes the weight of the fetus on the superior and inferior vena cava, promoting venous return and decreasing venous pressure. Choice C is incorrect. Increasing fiber and water intake promote the formation of bulkier stools. Preventing constipation and relieving rectal pain. Choice D is incorrect. Cold compresses relieve pain by vasoconstriction of the hemorrhoids.

An 11-week pregnant client is complaining to the nurse about her hemorrhoids. The nurse understands that hemorrhoids occur because of pressure on the rectal veins from the bulk of the growing fetus. All of the following are measures to alleviate hemorrhoid pain, except: A. Instruct the client to use mineral oil to soften her stools. B. Rest in a side-lying position daily. C. Increase the client's fiber and water intake. D. Apply a cold compress to the area.

D Choice D is correct. This patient's left pupil is fixed and dilated, which means it is not reactive to light and stays the same size. When this happens, it can be clinically inferred that there is a lesion or hemorrhage on the opposite (contralateral) side of the brain. The patient also exhibits decorticate (flexor) posturing, with elbows, wrists, and fingers flexed, while the legs are extended and rotated inward. Often, such abnormal posturing indicates severe brain damage. The patient sustained a fall and these symptoms likely represent raised intracranial pressure due to intracranial hemorrhage. This patient needs to be taken straight to the CT department to obtain a CT scan of the brain. This will allow the physician to diagnose the patient and initiate early treatment. Choice A is incorrect. Even though obtaining IV access is an important intervention, it is not the priority at this time. A non-contrast CT scan is usually the first intervention to detect a hemorrhage. Intravenous contrast is not necessary. The nurse can obtain IV access after the urgent CT scan is performed. Early diagnosis and appropriate treatment are critical in these settings. Choice B is incorrect. If the patient started having a seizure, then he would need to be turned onto his side. However, he is posturing, which is not a seizure. There are two different types of posturing; decorticate and decerebrate. Decorticate looks as if the patient is turning his or her arms into the core of the body. Decerebrate looks like the patient's arms are facing outwards, away from the body. Choice C is incorrect. This intervention is important, especially to understand any events before arriving at the hospital, medications taken, and recent procedures completed.

An 82-year-old man presents to the emergency department after a ground-level fall. The paramedics tell you that the left pupil was fixed and dilated. Upon arrival, the patient's elbows, wrists, and fingers are flexed, and legs extended and rotated inward. What is the most important intervention for this patient? A. Obtain IV access immediately B. Turn patient on his side C. Obtain accurate history from the family D. Take him straight to the CT scan

D Choice D is correct. First, determine if the ABG is compensated or uncompensated. Since the pH is not between 7.35 and 7.45, it is uncompensated. Next, decide whether it is acidosis or alkalosis. The pH is higher than 7.45, so it is an alkalosis. Lastly, determine if it is respiratory or metabolic. Evaluate the CO2 and HCO3 to see which is out of range. The HCO3 is normal, and the CO2 is low, so this is a respiratory problem. Putting it all together, this case is an uncompensated respiratory alkalosis. This patient is breathing off too much CO2, which is an acid. Because they are losing too much acid, they are becoming alkalotic, hence their pH of 7.62. The kidneys have not yet begun compensating by decreasing the amount of HCO3 they are producing, therefore the HCO3 value is still normal.

Analyze the following ABG: pH 7.62, CO2 19, HCO3 24 A. Compensated metabolic acidosis B. Uncompensated metabolic acidosis C. Compensated respiratory acidosis D. Uncompensated respiratory alkalosis

B Choice B is correct. You would document this assessment data as a #3 according to the Pasero Opiod-induced Sedation Scale (POSS). The POSS is a reliable tool used to assess sedation when administering opioid medications to manage pain. The Joint Commission and the American Society for Pain Management Nursing have endorsed the POSS to help prevent opioid-related adverse respiratory events. The scale is as follows: S: Asleep but easily aroused 1: Alert and awake 2: Sleepy but easily aroused 3: Arousable, but tired and falling asleep during conversations. This level is unacceptable. Monitor respiratory status closely until sedation level is #2 or lower. 4: Little or no response to physical stimulation. This level is unacceptable. Stop opioids and consider reversing with naloxone. Choice A is incorrect. A #2 according to the POSS, indicates that the client is sleepy but easily aroused. Choice C is incorrect. Coma is defined as complete unresponsiveness to all stimuli. Choice D is incorrect. Stupor is defined as the inability to respond to a stimulus other than vigorous stimulation.

Before administering an as-needed opioid for pain, the nurse assesses the client. It is determined that the client is arousable but tired and is falling asleep during conversation. How would the nurse best document this assessment data? A. A #2 according to the Pasero sedation scale B. A #3 according to the Pasero sedation scale C. Comatose D. Stuporous

B Choice B is correct. Inflammation from thrombophlebitis increases the size of the affected extremity and can be assessed by measuring circumference regularly. Thrombophlebitis is an inflammation of a vein associated with thrombus formation. Thrombophlebitis from venous stasis is most commonly seen in the legs of postoperative patients. Manifestations of thrombophlebitis are pain and cramping in the calf or thigh of the involved extremity, redness and swelling in the affected area, elevated temperature, and an increase in the diameter of the involved extremity. Each shift, nurses should assess the legs for swelling and tenderness, measure bilateral calf or thigh circumference, and determine if the patient experiences any chest pain or dyspnea. The patient should be instructed not to massage the legs. Choices A, C, and D are incorrect. These options are not the correct way to assess for the presence of thrombophlebitis.

How should the nurse assess for the presence of thrombophlebitis in a patient who reports having pain in the left lower leg? A. By palpating the skin over the tibia and fibula B. By documenting daily calf circumference measurements C. By recording vital signs obtained four times a day D. By noting difficulty with ambulation

C Choice C is correct. At the 8 to 10-week ultrasound; the radiologist will not be able to rule out all fetal abnormalities. Choices A, B, and D are incorrect. Ultrasounds performed during this period do help determine the due date as well as verify the pelvic shape and health.

The 8 to 10-week ultrasound verifies all of the following, except: A. Estimated Due Date (EDD) B. Pelvic shape C. The absence of fetal abnormalities D. Confirm pelvic health assessed at the first prenatal appointment

B Choice B is correct. This client is showing signs of heparin-induced thrombocytopenia (HIT): 50% decrease in platelets 5-10 days after heparin therapy was initiated. This is a thrombotic emergency and the nurse should assess the client, notify the physician, and discontinue the heparin drip. Choice A is incorrect. This client is being treated for pneumonia and is likely on antibiotics and corticosteroids. Both of these medications are known to increase blood glucose levels. This blood glucose result is high and the client may require a change in the insulin dose, but this would not be an emergency or the nurse's top priority. Choice C is incorrect. This client has an elevated A1C level (the ideal range is less than 7.0%). Hemoglobin A1C reflects blood sugar control over the past three months, so this would not be the highest priority. The patient complaining of tingling and numbness in the toes indicates peripheral neuropathy, a common problem in diabetic clients, mainly when blood sugars are poorly controlled. The nurse should determine what teaching/interventions the client needs to achieve better control of blood sugars and manage symptoms of neuropathy. Choice D is incorrect. This client is presenting with symptoms typical of acute post-streptococcal glomerulonephritis (APSGN): hypertension due to fluid retention, rust-colored hematuria due to upper urinary tract bleeding, and proteinuria due to decreased filtration. The symptoms that are expected are not the highest priority. Most clients with APSGN recover fully with conservative treatment and rest.

The RN is caring for clients on a med-surg unit. Which result would warrant immediate intervention by the nurse? A. A blood glucose level of 250 in a type 2 diabetic being treated for pneumonia. B. A patient on a heparin drip with a 50% decrease in platelets over the past week. C. A type 2 diabetic client with A1C 10.5 complaining of tingling and numbness in the toes. D. An acute post-streptococcal glomerulonephritis client with a BP of 140/88 mmHg, proteinuria, and rust-colored urine.

Stop the transfusion Administer oxygen Take vital signs Obtain a urine specimen

The nurse is caring for a patient receiving a blood transfusion. On assessment, the nurse notes that the patient's respirations are rapid, the face is flushed, and the patient is complaining of itching. The nurse suspects the patient is having a transfusion reaction. The nurse should accomplish the following actions: Take vital signs Stop the transfusion Administer oxygen Obtain a urine specimen. The nurse should complete the tasks in the following order: Stop the transfusion Take vital signs Administer oxygen Obtain a urine specimen

B Choice B is correct. An early ultrasound is the most accurate way to determine the estimated due date. One study found that birth occurred within seven days of the estimated due date determined by ultrasound alone. Choice A is incorrect. Nagele's rule is not the most accurate way to determine a prenatal client's due date. Choice C is incorrect. hCG levels vary from woman to woman and are not accurate in predicting a due date. Choice D is incorrect. Chadwick's sign can be used as a probable sign of pregnancy, but it does not help determine a due date.

The health care team is determining a prenatal client's estimated due date (EDD). Which of the following is the most accurate method used to determine the estimated due date? A. Nagele's Rule B. Embryonic ultrasound C. Early hCG levels D. Chadwick's sign

B, D, E Choices B, D, and E are correct. Anthrax is caused by a bacteria known as Bacillus anthracis. It is spread by inhaling bacterial spores, eating raw or contaminated meats, or through open wounds and scratches on the skin. Anthrax is not spread person to person or animal to person. Choices A and C are incorrect. These are not the way anthrax is spread.

The local community health nurse is teaching a course to nursing students on biological terrorism. When discussing anthrax, the nurse should inform their students that this agent is transmitted via: Select all that apply. A. Mosquito bites B. Breathing in bacterial spores C. Sexual contact with an infected person D. Ingestion of contaminated animal products E. Through an open wound or scratch on the skin

C Choice C is correct. A neologism is when a client invents words or phrases that only have meaning for themselves. This is a positive symptom associated with schizophrenia. Choices A, B, and D are incorrect. Words that rhyme or have a similar beginning sound are a clang association (example - 'She went to the bar, and saw some tar'). A reduction of speech with short-worded replies is alogia (poverty of speech). This is a negative symptom associated with schizophrenia. Going off on tangents and never reaching the point is the classic definition of tangentially. Schizophrenia symptoms are divided into positive or negative symptoms. Positive symptoms include things that add something to the client. They include: Hallucinations: Experiences involving the apparent perception of something not present. They can include any of the five senses: touch, taste, smell, sight, or hearing. Auditory hallucinations, when the client hears something that is not present, are common in schizophrenia. Delusions: Fixed, false beliefs that conflict with reality. Types of delusions include persecution, grandeur, and jealousy Thought and speech disorganization Negative symptoms are things that take something away from the client. They include: Apathy: A lack of interest, enthusiasm, or concern. Alogia: Also known as 'poverty of speech,' alogia is difficulty with speaking or the tendency to speak little due to brain impairment. Anhedonia: The inability to feel pleasure. Avolition: A total lack of motivation that makes it hard to get anything done Flattened affect

The nurse assesses a client with schizophrenia who appears to be demonstrating neologisms in their speech. Which of the following would be the expected finding? A. Words that rhyme or have a similar beginning sound B. Reduction in speech; short-worded replies C. Words or phrases with meaning only for the client D. Going off on tangents and never reaching the point

C Choice C is correct. The nurse should always confirm first whether another staff member returned the baby to the nursery. The nurse should not cause a false alarm in the institution. A Code Pink notifies all hospital staff of a possible infant abduction. Choice A is incorrect. When the nurse is sure that the infant is not in the nursery, a Code Pink can be started. This notifies all hospital staff of a possible infant abduction. Choice B is incorrect. This will be done if the infant was not returned to the nursery, but this is not the initial action of the nurse. Choice D is incorrect. There are many safety precautions to prevent infant abductions. For example, most facilities have a code word that is changed daily. The mother must ask anyone who wants to take the infant out of the mother's room for the code word. This is not the nurse's first intervention.

The nurse comes into the client's room to check on her and her newborn child. The client tells the nurse that another nurse just came and took the baby back to the nursery. What would be the initial action of the nurse? A. Alert security personnel about an infant abduction and call a code. B. Ask the mother what the nurse who took her baby looked like. C. Call the nursery to ask if the baby was returned to the nursery. D. Ask the mother if she asked the nurse for a code word.

A, F Choices A and F are correct. As people age, several physiological changes occur. Many of these changes impact the pharmacokinetics and pharmacodynamics of medications. The regular physiological changes associated with the aging process that can adversely affect the excretion and elimination of drugs in the human body are the aging population's low-functioning nephrons and diminished glomerular filtration. These changes can lead to the accumulation of medications in the body because they are not properly eliminated. Choices B, C, D, and E are incorrect. Decreased hepatic enzyme functioning slows down the metabolism of medications, but not the excretion and elimination of medications in the human body. Decreased peristalsis slows down the absorption of medications, but not the excretion and elimination of drugs in the human body. Increased pH of the gastric secretions, rather than lower pH, slows down the absorption of medications, but not the excretion and elimination of drugs in the human body. Increased alkalinity, not acidity, slows down the absorption of medications, but not the excretion and elimination of drugs in the human body. ✓ Older adults often have decreased renal blood flow, reduced kidney function, and decreased glomerular filtration rate, which all compound to the older adult having reduced elimination of a drug. ✓ Creatinine levels should be monitored because increased levels may suggest insult to the kidney by a nephrotoxic medication.

The nurse conducts a review course on older adults and medication elimination/excretion. It would be appropriate for the nurse to note which factor may impact drug elimination? Select all that apply. A. Diminished glomerular filtration B. Decreased enzyme functioning C. Decreased peristalsis D. Lower pH of the gastric secretions E. Increased acidity of the gastric secretions F. Low functioning nephrons

C Choice C is correct. In the total patient care model, the RN assumes responsibility for all aspects of care for a patient or group of patients during a shift, although care can be delegated. The RN works directly with the patient, family, and health care team members. Choice A is incorrect. With team nursing, the RN leads a team consisting of other RNs, LPNs, and NAPs, that provide direct patient care under the supervision of the team leader. Choice B is incorrect. Case management is an approach that coordinates health care services, linking patient/family to the needed services. Rather than providing direct patient care, the case manager typically supervises the care provided by the health care team. Choice D is incorrect. In primary nursing, the RN is assigned responsibility for a caseload of patients, developing the plan of care, and providing care for the workload of patients throughout their hospital stay.

The nurse has been assigned the responsibility for all aspects of providing patient care during a 12-hour shift. How would you classify this approach to nursing care? A. Team Nursing B. Case Management C. Total Patient Care D. Primary Nursing

A Choice A is correct. The Centers for Disease Control and Prevention cite handwashing as the single most effective way to prevent the transmission of disease. The effectiveness of other measures is dependent upon the foundation of appropriate hand hygiene. Choice B is incorrect. While many nurses do use exam gloves whenever in contact with patients, this will not be effective without practicing hand hygiene. Choice C is incorrect. While cleaning shared equipment will help prevent transmission of disease, it is not the most effective measure and is ineffective if the nurse had not practiced hand hygiene. Choice D is incorrect. Maintaining a distance of three feet from others who are coughing is recommended; however, this is not always feasible when providing patient care. Again, hand hygiene is the most effective preventative measure.

The nurse has been assigned to provide care for a group of patients that includes a patient with Mycoplasma pneumonia and a patient with Clostridium difficile diarrhea. What approach should the nurse use to best protect against the transmission of these infections to other patients? A. Perform hand hygiene before, after, and between providing direct patient care. B. Wear examination gloves whenever in direct contact with any patient. C. Cleanse equipment such as thermometers or stethoscopes between patients. D. Maintain a distance of 3 feet away from patients who are coughing.

C, D Choices C and D are correct. PrEP (pre-exposure prophylaxis) is helpful to those at risk for HIV to reduce infections. PrEP is highly effective in protecting a person from getting HIV from sex or injection drug use if taken as prescribed before the risky event. Combined antiretroviral therapy (cART) is the drug regimen indicated for HIV and AIDS. Maternal adherence to cART therapy may significantly reduce the risk of transmitting the disease to the fetus. Choices A, B, and E are incorrect. Surfaces contaminated with HIV/AIDS virus should be disinfected with a bleach product, not soap and hot water. The goal of cART is to increase the client's CD4 & CD8 counts and to decrease the viral load. Finally. HIV/AIDS is spread through contaminated needles, unprotected intercourse, and mother-to-child transmission (pregnancy, labor, delivery, and breastfeeding), not through contaminated water. ADDITIONAL INFO ✓ Human immunodeficiency virus (HIV) is a retrovirus that may lead to Acquire Immunodeficiency Syndrome (AIDS) if untreated ✓ Modes of transmitting HIV include - • Sexual: genital, anal, or oral (low risk) sexual contact with exposure of mucous membranes to infected semen or vaginal secretions • Parenteral: sharing of needles ("sharps") or equipment contaminated with infected blood or receiving contaminated blood products • Perinatal: from the placenta, from contact with maternal blood and body fluids during birth, or from breast milk from an infected mother to child. Pregnancy is a risk factor for HIV as the pregnant individual has engaged in unprotected copulation. HIV testing is routinely done in the third trimester ✓ HIV is not transmitted by Casual contact in the home, school, or workplace Sharing household utensils, towels, linens, and toilet facilities Mosquitos or other insects do not spread HIV ✓ HIV and AIDS treatment goals include increasing the CD4 count and reducing the viral load (VL) to undetectable ✓ PrEP (pre-exposure prophylaxis) is prescribed to clients with risk factors of contracting HIV

The nurse has just completed a continuing education lecture regarding the human immunodeficiency virus (HIV). Which of the following statements by the nurse indicate correct understanding? Select all that apply. A. "I will clean contaminated surfaces with soap and hot water." B. "The goal of treatment is for the client's viral load to increase and CD4 cells to decrease." C. "Pre-exposure prophylaxis (PREP) is available to those with risk factors for HIV." D. "Vertical transmission (mother to fetus) may be reduced with the use of antiretrovirals." E. "It is possible to spread the infection through contaminated water."

B Choice B is correct. The nurse's initial action should be to wash the area with soap and water thoroughly. The nurse should not squeeze the wound. Following this, the nurse should report the incident to the supervisor and complete the exposure report sheet. Choice A is incorrect. The incident should be documented; however, the nurse should care for the wound first. Promptly caring for the wound may minimize the risk of infection after exposure. Choice C is incorrect. Information should be obtained regarding the source of exposure (the client), however, caring for the needlestick wound is the priority. Subsequently, the client should be evaluated for HIV, HBV, and HCV status. The nurse may refer to the client's chart. If the status is unknown, the nurse should ask for consent and testing of these viruses. Choice D is incorrect. The major concern with needlestick injuries is viral transmission (HCV, HBV, and HIV), not bacterial infection. The first puncture site would not need an antibiotic ointment. Needlestick injuries are an occupational hazard for healthcare professionals. Nurses, surgeons, and laboratory workers are at the highest risk. If established procedures are followed, most needlestick injuries are preventable. Not following universal precautions, deviating from established safety protocols, and using needles without safety features are some reasons for needlestick injuries. Needlestick protocol: Should a needlestick injury occur, established protocol must be followed to minimize the risk of infection. Quick action within the established timeframe reduces the risk of infection transmission. Wash the wound thoroughly with soap and water Report to the supervisor and complete the incident report Assess the exposure (what type of needle and fluid? how much blood was on the needle?) Assess the client's status- refer to the client's chart and past history, and if the status is unknown, get consent to test the client for HBV, HCV, and HIV. Based on the client's status and risk of infection, proceed with appropriate post-exposure treatment/management

The nurse has just given an intradermal injection of PPD to a client in the clinic when she accidentally sticks herself in the finger with the used needle. What is the initial action of the nurse? A. Fill out an incidence occurrence report. B. Wash the area with soap and water right away. C. Ask the client if he has HIV or hepatitis. D. Put an antibiotic cream and bandage over the site.

A, C Choices A and C are correct. According to the National Asthma Education and Prevention Program Expert Panel Report 3, potential side effects of all the SABAs include tachycardia, headache, hypertension, hyperglycemia, tremors, hypokalemia, and increased lactic acid accumulation. The inhaled route is relatively safe since there are few systemic effects from the medication. Choices B and D are incorrect. Hypotension and hypoglycemia are not side effects of SABA use.

The nurse in the clinic is caring for a 10-year-old with asthma. The child uses an albuterol multi-dose inhaler before engaging in exercise. The nurse should educate the child and parents that potential side effects of this short-acting beta-2 agonist (SABA) are: Select all that apply. A. Tachycardia B. Hypotension C. Headache D. Hypoglycemia

C Choice C is correct. A complication of glomerulonephritis is encephalopathy caused by severe hypertension associated with the disease process. A client's report of a headache should clue the nurse into checking the client's blood pressure. The client should be monitored for this potential complication, which can be avoided by closely monitoring the client's blood pressure. Choices A, B, and D are incorrect. These manifestations are associated (and expected) with glomerulonephritis and do not require follow-up. AGN is a serious condition secondary to many infectious processes such as streptococcal infections, mononucleosis, and hepatitis. Nursing care aims to prevent the most common complications, including fluid volume overload and hepatic encephalopathy. The client may have dietary restrictions such as fluid, sodium, and potassium. The nurse should monitor the client's intake and output, weight, and blood pressure.

The nurse is assessing a child with glomerulonephritis. Which assessment finding requires follow-up by the nurse? A. Periorbital edema B. Decreased urine output C. Headache D. Hematuria

B Choice B is correct. The client is exhibiting signs of abruptio placentae. Diagnosis of abruptio placentae is predominantly based on clinical findings. The client may exhibit a sudden onset of abdominal pain or back pain. There is a sudden onset of mild to moderate vaginal bleeding in many cases. However, blood may get trapped inside the uterus in some cases, and there may not be any vaginal bleeding despite a severe abruption. Uterine contractions often come one right after another. The uterine tone increases, and therefore, the uterus feels hard/ rigid even in between the contractions. Based on the current international guidelines, the normal fetal baseline heart rate ( FHR) ranges from 110 beats per minute (bpm) to 160 bpm A heart rate of 99 bpm is significantly lower than the normal FHR. The patient is showing signs of abruptio placentae and an accompanying decreased fetal heart rate, indicating a fetus in compromise. If unaddressed, this issue will lead to fetal death. The concern for impending fetal demise should be prioritized. Choice A is incorrect. Pregnant women have fluid retention, resulting in electrolyte imbalances; this does not take priority over fetal demise. Choice C is incorrect. The fetus may experience altered gas exchange. There are no manifestations to suggest that the client is having any difficulty breathing; therefore, this would not be considered a problem for her. Choice D is incorrect. Pain relief for the mother should be a priority, but it should not overpower the potential death of the fetus.

The nurse is assessing a client in the labor and delivery department. She notes that the client's abdomen remains hard between contractions and that the fetal heart rate is 99 beats per minute (bpm). Which nursing diagnosis should take priority? A. Fluid and electrolyte imbalance B. Risk for fetal demise C. Ineffective breathing problem D. Alteration in comfort

A, C, D, E Choices A, C, D, and E are correct. Manifestations associated with appendicitis include leukocytosis, fever, nausea and vomiting, and anorexia. Choice B is incorrect. Melena is black tarry stools that occur because of gastrointestinal bleeding. This is not a feature of appendicitis. ✓ Appendicitis is an emergency that features pain in the right lower quadrant, nausea and vomiting, fever, leukocytosis, and anorexia. ✓ Appendicitis may be caused by obstruction, leading to inflammation and pressure. ✓ Nursing care for appendicitis includes - Maintaining nothing by mouth (NPO) status. Initiating an intravenous (IV) catheter. Administering prescribed antibiotics and IV fluids. Preparing the patient for surgery. The client should be monitored for perforation, which may manifest as abdominal pain that increases with cough or movement and is relieved by bending the right hip or the knees.

The nurse is assessing a client who has appendicitis. Which of the following would be an expected finding? Select all that apply. A. Leukocytosis B. Melena C. Fever D. Nausea and Vomiting E. Anorexia

B Choice B is correct. An S3 gallop is an expected finding in heart failure. This is often an early manifestation of heart failure; it and this sound are best auscultated at the apex of the heart. Choices A, C, and D are incorrect. Intermittent claudication is a clinical feature of peripheral arterial disease. This causes a client pain as they ambulate and is relieved with rest. The pain may radiate to the ankle or buttock. Venous stasis ulcers are associated with long-term venous insufficiency; ulcer formed due to edema or minor injury to the limb; typically occurs over the malleolus. A widened pulse pressure (difference between the systolic and diastolic) is a feature of increased intracranial pressure, not heart failure. A third heart sound, S3 gallop, is an early diastolic filling sound indicating an increased left ventricular pressure. This sound is often the first sign of HF. This sound is best auscultated over the apex of the heart. If this sound is auscultated during pregnancy, this is a normal finding.

The nurse is assessing a client with congestive heart failure. Which physical assessment finding should the nurse expect? A. Intermittent claudication B. S3 gallop C. Venous stasis ulcers D. Widened pulse pressure

C Choice C is correct. Phantom limb pain (PLP) is a form of neuropathic pain that can be treated with medications such as pregabalin, gabapentin, amitriptyline, or propranolol. Propranolol is a beta-adrenergic blocker, and while its action related to PLP is not fully understood, it has demonstrated efficacy for this type of pain. Choices A, B, and D are incorrect. Aripiprazole is an atypical antipsychotic and is not indicated in the management of PLP. Oxycodone is an opioid, and while opioids are effective for nociceptive pain, they have limited benefits for neuropathic pain. Hydroxyzine is a histamine antagonist commonly used in the treatment of allergies and anxiety. ADDITIONAL INFO Phantom limb pain (PLP) may occur after an above-the-knee amputation. This type of neuropathic pain is often described as a burning, crushing, or cramping sensation. It is essential for the nurse to acknowledge the pain and refrain from being dismissive (for example - telling the client that your limb is no longer present). Measures to treat PLP include medications such as pregabalin, gabapentin, amitriptyline, or propranolol.

The nurse is assessing a client with phantom limb pain following an above-the-knee amputation. The nurse anticipates a prescription for A. Aripiprazole B. Oxycodone C. Propranolol D. Hydroxyzine

A Choice A is correct. Neonatal manifestations of Hirschsprung's disease include failure to pass meconium, constipation, abdominal distention, and poor feeding. Choices B, C, and D are incorrect. Urinary retention, hematemesis, and a palpable abdominal mass are features not associated with Hirschsprung's disease. A palpable abdominal mass is a finding consistent with pyloric stenosis. Hirschsprung's disease is a congenital gastrointestinal disorder where the distal colon does not have functional ganglionic cells; therefore, the client does not have effective peristalsis. Several complications associated with this disease exist and include enterocolitis which may progress to toxic megacolon. Features of enterocolitis (fever, explosive and foul-smelling diarrhea, abdominal pain) should be reported immediately to the PHCP.

The nurse is assessing a neonate with Hirschsprung's disease. Which of the following would be an expected finding? A. Abdominal distention B. Urinary retention C. Hematemesis D. Palpable abdominal mass

D Choice D is correct. A retinal detachment is a medical emergency as it may become progressive and give the client blindness in the affected eye. The client may experience a loss of vision that appears as if a curtain is closing, or they may experience bright flashes of light. Choices A, B, and C are incorrect. A cloudy-like appearance in vision is a characteristic of cataracts. Intense pain above the eyebrow is a feature of angle-closure glaucoma, which is a medical emergency. This is when the intraocular pressure is quite high and gives a client symptoms such as eyebrow pain, eye redness, headache, and nausea. Alterations in peripheral vision are a characteristic of glaucoma. ADDITIONAL INFO When caring for a client with a suspected retinal detachment they should do the following - Encourage the client to have bed rest Apply an eye patch to both eyes Instruct the client to avoid sudden movements Prepare the client for surgery, as directed

The nurse is assessing a patient who has a suspected retinal detachment. Which of the following patient statements would be consistent with this diagnosis? A. "My vision has a cloudy appearance." B. "I have intense pain above my eyebrow." C. "I am having trouble with my peripheral vision." D. "I can see bright flashes of light."

C Choice C is correct. Bronchovesicular sounds occur over major bronchi where there are fewer alveoli. They are moderate in pitch and amplitude and are normally equal during inspiration and expiration. Posteriorly, bronchovesicular breath sounds can be auscultated between the scapulae. Choices A, B, and D are incorrect. The right second intercostal space is where the nurse may auscultate the aortic valve. The midclavicular line in the fifth intercostal space is the apex of the heart where the apical pulse may be obtained, and any adventitious sounds such as S3 or S4 may be auscultated. Auscultating lung sounds over the trachea is where the nurse may listen to bronchial breath sounds. These breath sounds are loud and high-pitched with hollow quality.

The nurse is auscultating bronchovesicular lung sounds on a client. The nurse understands that these lung sounds are best heard A. right second intercostal space. B. midclavicular line, in the fifth intercostal space. C. posteriorly, between the scapula. D. over the trachea.

D Choice D is correct. Avolition is a lack of motivation and is a key feature in schizophrenia as well as some depressive disorders. Avolition is categorized as a negative symptom associated with schizophrenia. Choices A, B, and C are incorrect. Loss of balance, full range of affect, and diminished expansion are not findings associated with avolition. Loss of balance would be a problem associated with proprioception. A diminished expression would be consistent with schizophrenia and other psychiatric disorders. This could be termed constricted or flat affect, depending on the significance. ADDITIONAL INFO Avolition is a clinical feature of some psychiatric illnesses, and this includes schizophrenia. This negative symptom may significantly impact a client's socioeconomic status as maintaining employment and social relationships may become difficult. The nurse should maintain the patient's activities of daily living and encourage participation.

The nurse is caring for a client demonstrating avolition. The nurse would expect to observe the client have which of the following? A. Loss of balance B. Full range of affect C. Diminished expression D. Lack of motivation

A Choice A is correct. Labor dystocia is a broad term indicating difficult labor that is not progressing. One of the medications that may be used to assist in labor progression is oxytocin. This medication stimulates uterine contractions. Choices B, C, and D are incorrect. Terbutaline is a tocolytic and is used to suppress uterine contractions. This medication would further decrease the progress of labor. Magnesium sulfate is a medication used in preterm labor to help stop contractions. It would be contraindicated in the client experiencing labor dystocia. Betamethasone is a steroid administered to mothers in preterm labor to help the development of the fetus's lungs in anticipation of preterm delivery. This medication is not indicated for labor dystocia. Labor dystocia describes difficult labor that does not progress as expected. During labor dystocia, uterine contractions are not effective or infrequent. Management of labor dystocia includes Administration of intravenous (IV) or oral fluids to correct any electrolyte imbalances. Frequent maternal position changes. Standing or sitting in a warm shower can be therapeutic. Pain management is important; however, epidural blocks decrease labor progress. Prescriptive treatments such as oxytocin may be used.

The nurse is caring for a client experiencing labor dystocia. Which medication does the nurse anticipate from the primary healthcare provider (PHCP)? A. Oxytocin B. Terbutaline C. Magnesium sulfate D. Betamethasone

B Choice B is correct. Early decelerations are a reassuring finding caused by infant head compression, which is a normal part of labor. Choices A, C, and D are incorrect. Repositioning the client and discontinuing an oxytocin infusion would all be appropriate interventions for a client experiencing variable or late decelerations. Early decelerations are a reassuring pattern and do not require the nurse to intervene. ✓ Early decelerations are not associated with fetal compromise and do not require intervention. ✓ Early decelerations occur during contractions as the fetal head is pressed against the client's pelvis or cervix.

The nurse is caring for a client in labor experiencing early decelerations. Which of the following actions should the nurse take? A. Reposition the patient on her side B. Document the findings C. Discontinue oxytocin infusion D. Prepare for an amnioinfusion

B Choice B is correct. Delirium Tremens (DTs) is a medical emergency that may result in seizure activity. The nurse should always put the client's safety at the forefront and provide seizure precautions. This includes padding the side rails, ensuring that intravenous access has been established, oxygen is at the bedside, and suction is available. Choices A, C, and D are incorrect. While assessing a client's pain is an essential task, this is not a pertinent assessment for DTs. The nurse should assure client safety by implementing seizure precautions. Obtaining a prescription for chlordiazepoxide (benzodiazepine) is a reasonable task but does not prioritize over assuring client safety. Benzodiazepines are the hallmark in preventing seizure activity in DTs and increasing the client's comfort during DTs (they experience tachycardia, hypertension, flushing, and diaphoresis). A GCS is not a relevant assessment for DTs, as The Clinical Institute, Withdrawal Assessment Alcohol Scale-Revised (CIWA-Ar) is utilized to determine the severity of the withdrawal. ✓ Delirium Tremens (DTs) are a medical emergency and may cause autonomic hyperactivity, resulting in tachycardia, diaphoresis, fever, anxiety, insomnia, and hypertension. ✓ Delusions and visual and tactile hallucinations are common in alcohol withdrawal delirium. ✓ This may occur within 72 hours following the last alcoholic beverage consumed. ✓ Withdrawal seizures may occur within 12 to 24 hours after alcohol cessation. ✓ These seizures are generalized and tonic-clonic. Additional seizures may occur within hours of the first seizure. ✓ Diazepam is given intravenously as a common treatment for withdrawal seizures. Nursing care for DTs includes - Rapid assessment of the client's vital signs Initiate seizure precautions and establish patent intravenous access Obtain a prescription for benzodiazepines, such as lorazepam or diazepam Administer intravenous fluids and electrolytes to replete the lost fluids Assess the client using the CiWa-Ar scale to trend the severity of the symptoms

The nurse is caring for a client in the emergency department (ED) experiencing delirium tremens. The nurse should take which initial action? A. Assess the client's pain level. B. Implement seizure precautions. C. Obtain a prescription for chlordiazepoxide. D. Administer the Glasgow Coma Scale (GCS).

11 mL/hr

The nurse is caring for a client prescribed IV heparin. The client is prescribed 12 units/kg/hr. The client weighs 92 kg. The heparin is labeled with 25,000 units in 250 mL of D5W. How many mL/hr should this client receive? Round your answer to the nearest whole number. Fill in the blank.

11 mL/hr

The nurse is caring for a client prescribed IV heparin. The client is prescribed 15 units/kg/hr. The client weighs 154 pounds. The heparin is labeled with 25,000 units in 250 mL of D5W. How many mL/hr should this client receive? Round your answer to the nearest whole number. Fill in the blank.

A Choice A is correct. Treatment goals for a patient with Polycystic Kidney Disease (PKD) include maintaining normotension, the glomerular filtration rate (GFR), and the prevention of sodium wasting, which is evidence of a decline in renal function. Hypertension is a cardinal finding in PKD, and if a client is achieving the treatment goals, they will maintain regulated blood pressure. Choices B, C, and D are incorrect. An elevated creatinine indicates that kidney function is declining. Proteinuria and sodium in the urine are also evidence of decreased renal function. Both findings would not indicate that the client is improving; rather, they would indicate that the client's renal function is declining. Polycystic Kidney Disease is a genetic disorder manifested by fluid-filled cysts that grow on the kidneys. Additional findings in PKD include: • Abdominal or flank pain • Hypertension • Nocturia • Frequent urinary tract infections • Increased abdominal girth • Constipation • Hematuria (bloody urine) • Sodium wasting and inability to concentrate urine in the early stage • Progression to kidney failure with anuria

The nurse is caring for a client who has polycystic kidney disease (PKD). Which of the following would indicate the client is achieving treatment goals? A. Blood Pressure 128/63 mmHg B. Creatinine 2.3 mg/dL C. Proteinuria 2+ D. Sodium 132 mEq/L

B Choice B is correct. Rubella is known as German measles and requires droplet precautions. The nurse is right to wear a surgical mask when engaging with the client. The transmission mode for rubella is a droplet mode of communication where the spread occurs with particle drops larger than 5 microns. Choices A, C, and D are incorrect. Do not confuse rubeola with rubella. Rubeola (measles) requires airborne precautions, whereas rubella needs droplet precautions. Airborne transmission occurs when the pathogen is carried in dust or droplets in the air that remains in place for sufficient time to infect a person exposed to this air. Contact transmission precautions are used for infectious diseases spread by contact with the client or the client's environment. Contact transmission precautions prevent infections such as C. difficile and shigella, but not rubella. Standard precautions are a set of infection control practices used to prevent the transmission of diseases that can be acquired by contact with blood, body fluids, non-intact skin (including rashes), and mucous membranes. Droplet precautions involve wearing a surgical mask and are used for rubella. Standard precautions do not include routinely wearing a surgical mask and are insufficient to prevent rubella transmission. Personal Protective Equipment (PPE) required for a client with droplet precautions is a surgical mask. The risk of transmitting a droplet pathogen is likely when the nurse is within three feet of the individual infected. Conditions requiring droplet precautions include: Rubella Influenza Pertussis Bacterial meningitis Pneumonic plague Diphtheria (Pharyngeal) Mumps Rhinovirus

The nurse is caring for a client who has rubella. The nurse should isolate the client using which of the following? A. Airborne precautions B. Droplet precautions C. Contact precautions D. Standard precautions

B Choice B is correct. Enalapril is an ACE inhibitor used to treat hypertension and congestive heart failure (CHF). The priority teaching would be about the adverse reaction of angioedema (swelling of the face, lips, eyes) because it may lead to respiratory distress. Choices A, C, and D are incorrect. ACE inhibitors may cause a dry, non-productive cough, alterations in taste, and hyperkalemia. However, the essential teaching point is about angioedema's adverse reaction, which could be life-threatening if left untreated. ADDITIONAL INFO Angiotensin-Converting Enzyme Inhibitors (ACE-I) include medications such as (lisinopril, captopril, and enalapril). These medications are prescribed for hypertension, congestive heart failure, and diabetic nephropathy. A dry cough is common with ACE inhibitors. Hyperkalemia may occur with an ACE inhibitor, and potassium supplementation or potassium-sparing diuretics should be used with caution. Like all ACE inhibitors, lisinopril can cause a dry cough, which is annoying and will not harm the client. Lisinopril (and all ACE inhibitors) may be associated with a decrease in renal function. Thus, the creatinine should be monitored during the therapy.

The nurse is caring for a client who is receiving prescribed enalapril. Which of the following would be essential to teach the client? The client should immediately report A. dry non-productive cough. B. swelling of the face, lips, and eyes. C. alterations in taste. D. the need for follow-up laboratory work.

C Choice C is correct. Oprelvekin is a hematopoietic agent used to stimulate the production of platelets. This platelet count is normal (150-400 mm3 is the optimal range) and, thus, is a therapeutic finding. Choices A, B, and D are incorrect. Oprelvekin is a hematopoietic agent used to stimulate the production of platelets. While all of these laboratory values are normal, they are not relevant to the therapeutic benefit of oprelvekin. ADDITIONAL INFO ✓ Oprelvekin is indicated to prevent chemotherapy-induced severe thrombocytopenia and avoid the need for platelet transfusions. ✓ This medication takes five to nine days to be efficacious. ✓ Once oprelvekin is discontinued, counts remain increased for about seven days and return to baseline within fourteen days. ✓ The normal platelet count is 150,000-400,000 mm3

The nurse is caring for a client who is receiving prescribed oprelvekin. Which of the following client findings would indicate a therapeutic response? A. Hemoglobin (Hgb) 14 g/dL B. White Blood Cell (WBC) 6,500 mm3 C. Platelets 155,000 mm3 D. Prothrombin Time (PT) 11 seconds

B Choice B is correct. Most Jehovah's witnesses do not condone the use of blood products and often refuse blood transfusions. This nurse should verify this information in the chart and with the client. Most Jehovah's Witnesses believe a human must not sustain his life with another creature's blood. They believe Jehovah will turn its back on anyone receiving blood products. Choice A is incorrect: Members of the Catholic faith have no set doctrines that prohibit the administration of blood products. Choice C is incorrect: Concerning the Islamic faith, Muslims are allowed to participate in blood and organ donation in cases of necessity and/or if the donation will save another person's life. This mean's that in life-or-death situations a blood transfusion is allowable for members of the Islamic faith. Choice D is incorrect: The Christian Reformed Church believes that their salvation is a gift from God and that good works are the Christian response to the gift. None of their doctrines prohibit the use of blood transfusion or blood product donation. ✓ Blood transfusions are commonly declined by individuals who engage in the Jehovah's Witnesses religion. ✓ This includes autologous blood donation, as this religion believes blood should not be taken out of the body. ✓ Albumin and certain coagulation concentrates are generally accepted because they are present at birth and cross the placenta.

The nurse is caring for a client who may require a prescribed blood transfusion. The nurse understands which of the following cultural groups prohibit blood transfusions? A. Catholicism B. Jehovah's witnesses C. Islam D. Christian reform

A, B, D, E Choices A, B, D, and E are correct. Increased urinary output, tachycardia, and orthostatic hypotension are expected findings with hyperglycemia. Blurred vision: hyperglycemia causes alteration of the eye lens, which explains the blurred vision. Polyuria: Increased urine output due to glycosuria. Glycosuria causes osmotic diuresis - glucose in the urine pulls excessive water with it, resulting in significant water losses and subsequent dehydration. Tachycardia: increased heart rate from the diuresis leading to dehydration. Orthostatic hypotension: Postural (orthostatic) hypotension is defined as a drop in systolic blood pressure of at least 20 mm Hg or more and diastolic blood pressure of at least 10 mm Hg or more within two to five minutes of quiet standing after five minutes of supine rest. Symptoms of hyperglycemia include increased thirst (polydipsia), polyuria, polyphagia, weight loss, blurry vision, and slow wound healing. Long-standing hyperglycemia can lead to nerve damage resulting in neuropathy (tingling, numbness, neuropathic pain). Hyperglycemia leads to osmotic diuresis when glucose levels are so high that glucose is excreted in the urine. Water follows the glucose concentration passively, leading to abnormally high urine output. In turn, this leads to dehydration. Dehydration manifests with tachycardia because the body responds to maintain perfusion by increasing cardiac output. Dehydration results in hypovolemia, which may present with orthostatic hypotension. Choice C is incorrect. Cold and clammy skin are expected findings with hypoglycemia, not hyperglycemia.

The nurse is caring for a client who presents with hyperglycemia. Which of the following findings are expected? Select all that apply. A. Blurred vision B. Increased urinary output C. Cool and clammy skin D. Tachycardia E. Orthostatic hypotension

A Choice A is correct. When caring for a client with a significant thermal burn (greater than 10% TBSA), the priority is assessing respiratory status. Smoke inhalation injuries and carbon monoxide poisoning are immediate concerns that must be addressed. Choices B, C, and D are incorrect. Fluid resuscitation is a central part of the emergent care of a major thermal burn. An isotonic solution, such as lactated ringers, is commonly utilized because of its fluid volume expanding properties. An indwelling catheter is necessary to determine the effectiveness of fluid resuscitation. However, this intervention does not prioritize airway patency. An accurate weight is necessary to determine the amount of fluid needed to restore circulating volume. The emergent (resuscitation) phase of a burn injury begins at the onset of injury and continues for about 24. The type of burn determines the type of care along with the percentage of body surface affected. The priorities of care during the emergent phase include: Securing the airway Supporting circulation and perfusion through volume repletion Maintaining body temperature Providing adequate pain control Rendering emotional support

The nurse is caring for a client who sustained full-thickness burns to their entire torso and back. The nurse plans to take which priority action? A. Assess the client's respiratory status B. Prepare an infusion of lactated ringers C. Insert an indwelling urinary catheter D. Obtain an accurate weight

B Choice B is correct. A bronchodilator would benefit this client as the arterial blood gas demonstrates respiratory acidosis. The accumulation of CO2 causes respiratory acidosis, and a bronchodilator such as formoterol or albuterol would help with exhaling the excessive CO2. Choices A, C, and D are incorrect. This client is oxygenating just fine, and supplemental oxygen would be inappropriate. The normal PaO2 is 80 - 100 mm Hg, and the normal oxygen saturation is 95% or greater. Regular insulin would be a treatment for metabolic acidosis that is caused by diabetic ketoacidosis, this is not shown in the ABG. Sodium polystyrene is a medication that treats hyperkalemia that would not be found with respiratory acidosis. The causes of respiratory acidosis include pulmonary emphysema, atelectasis, and hypoventilation. Treatment is aimed at the underlying cause, including instructing the client to turn, cough, and breathe deeply. An incentive spirometer may also be used to help treat respiratory acidosis.

The nurse is caring for a client with an arterial blood gas (ABG) of pH = 7.33; PaO2 = 95 mm Hg; PaCO2 = 53 mm Hg; HCO3- = 24 mEq/L; SaO2 = 96%. Which prescription should the nurse request from the primary healthcare provider (PHCP)? A. Supplemental oxygen B. Bronchodilator C. Regular insulin D. Sodium polystyrene

A, C, D Choice A, C, and D are correct. An ectopic pregnancy is a gynecological emergency. If not treated, a rupture may occur that leads to intraperitoneal bleeding. Classic manifestations of an ectopic pregnancy include: Presumptive pregnancy signs (nausea, breast tenderness) Vaginal bleeding Increased human chorionic gonadotropin (hCG) levels causing a positive pregnancy test Unilateral abdominal pain that may be confined to the pelvic area Choices B and E are incorrect. An ectopic pregnancy (EP) is not an infectious process like appendicitis. So fever is not an expected finding. Finally, dysuria would be a manifestation associated with cystitis. Again, an EP is not infectious and would not cause dysuria. An ectopic pregnancy (EP) is an extrauterine pregnancy. Almost all ectopic pregnancies occur in the fallopian tube, but other possible sites include cervical, interstitial, hysterotomy (cesarean) scar, ovarian, or abdominal. Manifestations of an EP include unilateral abdominal (pelvic) pain, vaginal bleeding, and a positive pregnancy test. Rapid management is needed because life-threatening intraabdominal bleeding may occur. For EP's that have not ruptured and the woman is stable, methotrexate may be used. If that is not the case, surgical management will be necessary.

The nurse is caring for a client with an ectopic pregnancy. Which findings does the nurse expect? Select all that apply. A. Pelvic pain B. Fever C. Vaginal bleeding D. Positive pregnancy test E. Dysuria

B Choice B is correct. Modafinil is a psychostimulant that is effective in treating narcolepsy. This medication promotes wakefulness and is dosed either once or twice a day. Choices A, C, and D are incorrect. Aripiprazole and quetiapine are atypical antipsychotics and not indicated in the management of narcolepsy. Quetiapine is highly sedating and would be counterproductive in the management of narcolepsy. Ropinirole is a dopaminergic and is indicated in treating Parkinson's disease and restless leg syndrome. Narcolepsy is a syndrome in which a client has significant daytime sleepiness that often lessens after a nap. Stimulants such as modafinil and armodafinil are indicated in the management of narcolepsy as they promote wakefulness. Common side effects of modafinil include headache, nervousness, anorexia, and weight loss.

The nurse is caring for a client with narcolepsy. Which of the following medications would the nurse anticipate the primary healthcare provider (PHCP) prescribe? A. Aripiprazole B. Modafinil C. Ropinirole D. Quetiapine

C Choice C is correct. Sumatriptan is a 5-hydroxytryptamine (5-HT))-receptor agonist indicated for abortive migraine headache treatment. Sumatriptan stops headaches after they have begun ( abortive therapy), but it does not prevent them. Choices A, B, and D are incorrect. These conditions are all contraindications to the administration of this medication. Sumatriptan has been linked to the development of sudden myocardial infarction following its administration in at-risk individuals. Sumatriptan may induce vasoconstriction. A client with severe hypertension, coronary artery disease, history of stroke, and peripheral vascular disease should not take this medication. Sumatriptan is a medication indicated to abort migraine headaches. It is not a prophylactic treatment. Sumatriptan is commonly administered intranasally or subcutaneously. The nurse should educate the client that if they receive this medication as an injection, a warm and tingling sensation is normal and temporary. Contraindicated for clients with uncontrolled hypertension, coronary artery disease, and those who have sustained a stroke or myocardial infarction. The nurse should monitor the client's blood pressure, autonomic instability, altered mental status, and manifestations similar to serotonin syndrome. Common side effects include flushing, tingling, warmth, dizziness, gastric upset, nausea, and vomiting.

The nurse is caring for a client with newly prescribed sumatriptan. The nurse understands that this medication is intended to treat which condition? A. Peripheral artery disease B. Accelerated hypertension C. Migraine headache D. Angina

A, B Choices A and B are correct. ( Choice A) Generally, residuals over 150 mL are considered above-normal volumes, although there is no need to withhold feeding for gastric residual volume (GRV) less than 500ml. The nurse should auscultate bowel sounds to detect potential signs of delayed gastric emptying in a patient with a large residual. According to a research study, the gastric emptying delay is classified into three levels based on the amount of residual volume - an amount of more than 150mL up to 250 mL is considered a mild delay. In contrast, amounts greater than 350 mL are classified as a severe delay. Per the American Society of Parenteral and Enteral Nutrition (ASPEN) and many other critical care society guidelines, the nurse should not hold feeding for a GRV of less than 500 mL in the absence of any clinical signs of intolerance. Therefore, the nurse should auscultate for bowel sounds and assess any nausea, emesis, or abdominal distention. Should there be any clinical signs of intolerance, the nurse should hold the feeding. When a large-volume residual is aspirated, it may indicate delayed gastric emptying. A prokinetic agent can be given to enhance gastric emptying. ( Choice B) The nurse should document the color, odor, consistency, and amount of the residual. One can remember this documentation from the mnemonic COCA: color, odor, consistency, and amount. For example, the nurse can document the aspirate as bright green, non-odorous, thin, 250 mL residual in this case. Such documentation will help establish a baseline for the client and identify changes that could indicate a concern. Choice C is incorrect. There is no information regarding feeding intolerance in the question stem. In the absence of feeding intolerance, the nurse should not discard the residual if it is less than 250 ml. Residuals up to 250 mL can be safely returned to the client. The nurse has aspirated stomach contents rich in electrolytes such as potassium and chloride. If the nurse discards the residual, the electrolytes will be lost, potentially exacerbating the imbalance. Choice D is incorrect. Guidelines recommend holding tube feeding only when the residual is greater than 500 mL or in the presence of any clinical signs of feeding intolerance. It is premature not to administer tube feeding without auscultating the bowel sounds and checking for signs of intolerance. If there are no signs of intolerance, the nurse should return this residual to the client, flush the tube with saline to ensure it remains patent, continue the tube feeding, and notify the physician.

The nurse is caring for a critically ill client receiving enteral nutrition through a nasogastric tube. Before initiating the next bolus feed, the nurse checks the residual and notes 225 mL of bright green fluid. Which actions by the nurse are appropriate? Select all that apply. A. Auscultate for bowel sounds B. Document the residual C. Discard the residual D. Do not administer the tube feeding

B Choice B is correct. Semi Fowler's position is the most appropriate position after a hyphema, or blood in the anterior chamber has been diagnosed. This position works with gravity to keep blood accumulation away from the optical center of the cornea. Choice A is incorrect. Supine, or lying facing upwards, is not the best position to place a patient who is experiencing hyphema after a car accident. This position could cause blood to accumulate near the optical center of the cornea. Choice C is incorrect. Lateral-lying, whether on the affected or unaffected side, does not keep blood from collecting near the optical center of the cornea. Choice D is incorrect. Lateral-lying, whether on the affected or unaffected side, does not keep blood from collecting near the optical center of the cornea.

The nurse is caring for a patient in the emergency department who has just received a head injury following a car accident. After a hyphema has been noted, which position should the nurse encourage this patient to be in? A. Supine B. Semi-Fowler's C. Lateral on the affected side D. Lateral on the unaffected side

B Choice B is correct. Ringing in the ears or tinnitus is the most common complaint from people with inner ear disorders. Choice A is incorrect. Itchiness is not the most common complaint in patients with disorders of the inner ear. Instead, tinnitus (ringing of the ears) is a more common complaint. Choice C is incorrect. Hearing loss is not the most common complaint in patients with disorders of the inner ear. Instead, tinnitus (ringing of the ears) is a more common complaint. Choice D is incorrect. A burning sensation in the ear is not the most common complaint in patients with disorders of the inner ear. Instead, tinnitus (ringing of the ears) is a more common complaint.

The nurse is caring for a patient with a disorder of the inner ear. Which of the following is the most common complaint about patients with these disorders? A. Itchiness B. Ringing in the ears C. Hearing loss D. A burning sensation in the ear

B, C, D Choices B, C, and D are correct. Late decelerations are a non-reassuring fetal heart pattern that requires immediate intervention. The nurse should place the client in a left lateral position, administer oxygen via face mask, and discontinue oxytocin. Repositioning the client will relieve the vena cava compression. Oxygen will correct any maternal hypoxia, and oxytocin should be stopped because uterine contractions decrease uteroplacental blood flow. Choices A and E are incorrect. Magnesium sulfate is indicated for clients with preeclampsia or eclampsia. It has no use for late decelerations. Amnioinfusion of an isotonic solution is utilized for variable decelerations - not late decelerations. ✓ Late decelerations are primarily caused by uteroplacental insufficiency, and the client should be repositioned into a left lateral position. ✓ If the left lateral position is ineffective, the nurse may consider using the right lateral position. ✓ The nurse should also consider prescribed intravenous (IV) fluids to restore maternal blood volume.

The nurse is caring for a pregnant client who is experiencing late decelerations. Which of the following actions should the nurse take? Select all that apply. A. Initiate intravenous magnesium sulfate B. Reposition the patient on her left side C. Administer oxygen via face mask D. Discontinue oxytocin infusion E. Prepare for an amnioinfusion

A Choice A is correct. For a preschooler experiencing pain resulting from a tonsillectomy, give the child a "magic" blanket to take the pain away. Preschoolers are magical and mystical thinkers, so this "magic" blanket may be an effective pain management technique for children of this age. Choices B, C, and D are incorrect. Hot soup is not recommended following a tonsillectomy because of its significant irritation to the affected areas. Privacy would not especially mitigate the client's pain at this age; instead, the presence of the parent(s) and the nurse may help to relieve the pain through distraction. Preschoolers cannot develop better coping skills in terms of pain because they are not mature enough to develop these particular coping skills. For the hospitalized preschooler, the nurse should plan the following interventions: ✓ Provide choices as much as possible ✓ Provide the child with their favorite objects or toys ✓ Using a routine for bedtime and morning rituals helps decrease anxiety ✓ Preschoolers have a magical thought process, and using strategies such as a magical blanket may help mitigate pain

The nurse is caring for a preschooler following a tonsillectomy. Which intervention would be appropriate for the nurse to implement? A. Place a "magical" blanket on the client to take the pain away. B. Offer hot soup in the toddler's favorite cup C. Provide the client with privacy so they are not embarrassed about crying. D. Assist the client in developing better coping skills for their pain.

C Choice C is correct. Immediately following a surgical procedure, the client is at risk for complications, including shock. The nurse should assess vital signs to determine a client's stability. The other assessments are not as essential because they do not help determine the hemodynamic stability of the client. Choices A, B, and D are incorrect. Following a total hip arthroplasty, the client will work with physical therapy to guide them on using ambulatory aids and completing daily activities. The range of motion is not a critical assessment, nor is the incision. Urine output must be assessed because older adult clients are sensitive to the anticholinergic effects found in certain anesthesia. Immediately following a surgical procedure (such as total hip arthroplasty), immediate assessments should include: Vital signs Lung sounds Cardiac rhythm Incision status Pain Nausea and vomiting Older adults are more at risk for complications, including suboptimal thermoregulation, urinary retention, wound disruption, and delirium.

The nurse is caring for an 88-year-old client who is fifteen minutes post-operative following total hip arthroplasty. It would be a priority to assess which of the following? A. Range of motion B. Urine output C. Vital signs D. Incision

A Choice A is correct. A trochanter roll placed on the outer aspect of the thigh holds the hip in a neutral position and keeps the leg in normal alignment. A trochanter roll refers to a towel/blanket rolled up into a cylindrical prop and placed around the lateral/hip thigh area extending from above the hip to above the knee. When the patient is supine, this trochanteric roll prevents external rotation and decreases the chance of further tissue damage or hip dislocation. The soft tissues and muscles around the hip are weak following surgery so, patients are instructed to follow "standard hip precautions" to decrease the chance of hip dislocation following total hip arthroplasty (THA). Choice B is incorrect. Range of motion exercises would not prevent or address the external rotation of the leg. Choice C is incorrect. Internal rotation of the leg is not beneficial to the client and may cause more injury. Normal alignment is required, and the pin must be in a neutral position. Choice D is incorrect. Telling the client to stay in a hip-neutral position may not help. The leg will come back to the external rotation once the client becomes unaware of the situation. The pin needs to be propped up in proper alignment with a trochanter roll.

The nurse is caring for an elderly patient who is status-post total hip replacement surgery. The nurse notices that the patient's left leg is externally rotated. Which of the following should the nurse do next? A. Place a trochanter roll on the outer aspect of the thigh B. Perform a range of motion exercises on the left leg C. Internally rotate the left leg D. Tell the client to keep his leg in a neutral position

A, E Choices A and E are correct. A client with diabetic ketoacidosis (DKA) receiving regular insulin intravenously is at significant risk for hypoglycemia because regular insulin (given intravenously) peaks within fifteen to thirty minutes. This is why the client has their glucose taken every hour. Lispro insulin is a rapid-acting insulin, and if the client has not eaten within ten to fifteen minutes of getting the insulin, they run the risk of hypoglycemia. Choices B, C, and D are incorrect. Methylprednisolone is a corticosteroid that causes hyperglycemia. This client would need more aggressive glucose control while they are receiving this medication. TPN may cause hyper- or hypoglycemia; however, the risk of hyperglycemia is much higher, especially if the client has pancreatitis. The client who has had a CABG will likely have hyperglycemia because of the stress associated with the surgery. Following major surgery, the adrenal glands discharge cortisol which raises blood glucose. It will be important for the client to have good post-operative glucose control, which is associated with better outcomes. Hypoglycemia may be caused by - Inappropriate dosing of insulin or antidiabetic medications such as glipizide Insulin and mealtime mismanagement (example - rapid-acting insulin given when the client is NPO or given too early prior to a meal) Inappropriate dosing of insulin Exercise or the consumption of alcohol

The nurse is caring for assigned clients. The nurse should recognize which client is at risk of developing hypoglycemia? Select all that apply. A client A. with diabetic ketoacidosis receiving continuous regular insulin intravenously. B. receiving methylprednisolone for an exacerbation of asthma. C. with pancreatitis and is receiving total parenteral nutrition (TPN). D. who is nothing by mouth (NPO) status following a coronary artery bypass graft (CABG). E. who received six units of lispro insulin one hour ago and has not eaten.

B Choice B is correct. This client has the highest risk for osteoporosis because of her post-menopausal status and smoking cigarettes. While the client has stopped drinking alcohol, the client's other risk factors of being female, post-menopausal, and tobacco abuse are significant. Choices A, C, and D are incorrect. Jogging would be a mitigating factor for osteoporosis, and the client's age, 27, is insignificant for osteoporosis. For women, osteoporosis is likely after the age of 50. The only risk factor this client has is alcohol exposure. The client is 51, and male is a low-risk factor compared to the female gender. Osteoporosis typically peaks for men at 70 when testosterone levels decline. Alcohol exposure is a risk factor for osteoporosis. The client stopping smoking is a protective factor for osteoporosis. The age of 25 is not a risk factor for osteoporosis. The only risk factor that this client has is corticosteroid usage. ✓ Osteoporosis is a condition characterized by a decrease in bone mass ✓ Risk factors are postmenopausal women, Asian or White race, excessive use of alcohol, tobacco use, sedentary lifestyle, prolonged exposure to corticosteroids, hormone insufficiency (testosterone/estrogen) ✓ The biggest complication of osteoporosis is fracture ✓ Prevention is the best treatment by encouraging weight-bearing exercises to increase bone mass, adequate calcium and vitamin D intake, and smoking, and alcohol cessation ✓ Diagnosis is established by a bone density (DEXA) scan ✓ Nursing care is focused on mitigating risk factors, preventing falls, and encouraging the client to remain active to help with restoring bone mass

The nurse is conducting a community health fair. Which of the following clients is at the greatest risk of developing osteoporosis? A. 27-year-old woman who jogs three times a week and drinks red wine daily B. 60-year-old woman who smokes cigarettes and just recently stopped drinking alcohol C. 51-year-old man who suffers from alcoholism and recently stopped smoking D. 25-year-old man with asthma who takes inhaled corticosteroids

A Choice A is correct. Hand hygiene is a crucial component of standard precautions. It is the most important measure to prevent the spread of infections among clients. Usually, hand hygiene may be performed either by washing hands with soap and water or by using an alcohol-based sanitizer. However, in certain situations, it is required that hand hygiene is performed only by washing hands with soap and water. One of these situations is when the hands are visibly soiled. Choices B, C, and D are incorrect - Alcohol-based sanitizers may be used when the hands are not visibly soiled. In situations such as collecting vital signs, performing range of motion exercises, and inputting data into the electronic medical record (EMR) it is absolutely reasonable to use santizers because the risk of coming into contact with blood or bodily fluids that may contaminate hands is quite low. According to the Centers for Disease Control and Prevention: Unless hands are visibly soiled, an alcohol-based hand rub is preferred over soap and water in most clinical situations due to evidence of better compliance compared to soap and water. Hand rubs are generally less irritating to hands and, in the absence of a sink, are an effective method of cleaning hands.

The nurse is conducting a staff conference regarding standard precautions. It would be correct for the nurse to state that hand washing with soap and water is required when A. hands are visibly soiled. B. collecting vital signs (VS). C. performing range of motion exercises. D. inputting data into the eletronic medical record (EMR)

A Choice A is correct. Following bariatric surgery, the client faces various complications, including hemorrhage, wound disruption, pneumonia, and infection. Venous thromboembolism is a significant complication and may be mitigated using pneumatic compression devices as well as chemical prophylaxis. Choices B, C, and D are incorrect. Insertion of a urinary catheter is not standard of care for a client recovering from bariatric surgery. This could cause bacterial cystitis, which may complicate healing. Strict bed rest is not recommended because this increases the risk for pneumonia and venous thromboembolism. Patients are often expected to ambulate several hours after surgery. Measuring the abdominal girth is an intervention for a client with ascites and is not applicable to those who have undergone bariatric surgery. ✓ Bariatric surgery is major surgery with many complications, such as venous thromboembolism, nutritional deficiencies, dumping syndrome, pneumonia, and infection. ✓ Early ambulation, compression devices, and chemical prophylaxis may mitigate venous thromboembolism. ✓ It is common for those who have undergone this surgery to ambulate within six to eight hours post-operatively to prevent such complications.

The nurse is developing a plan of care for a client who had bariatric surgery. Which of the following should the nurse include? A. Pneumatic compression devices B. Insertion of an indwelling urinary catheter C. Strict bed rest D. Measure the abdominal girth

D Choice D is correct. Cushing's disease produces elevated cortisol levels. Cortisol is best known for helping support the body's natural "fight-or-flight" instinct in a crisis. It also plays a vital role in several other body functions, including managing the use of carbohydrates, fats, and proteins, regulating blood pressure, increasing blood sugar levels, controlling the sleep/wake cycle, and boosting energy to help manage stress and restore balance. Choice A is incorrect. Addison's disease produces decreased cortisol levels. Choices B and C are incorrect. Neither of these conditions is associated with cortisol levels.

The nurse is educating a new graduate about alterations in cortisol levels. Which of the following conditions does she explain cause an increased cortisol levels in a client? A. Addison's disease B. Congestive heart failure C. Renal failure D. Cushing's disease

B Choice B is correct. Normal saline is the most appropriate intravenous fluid for blood transfusions. 0.9% saline (normal saline) is an isotonic solution and will not cause red blood cell (RBC) hemolysis or clumping. Choices A, C, and D are incorrect. Lactated Ringers (LR) can cause RBC clumping and hemolysis. Additionally, LR has calcium that promotes blood clotting. Therefore, LR is not a preferred fluid with blood transfusions. 0.45% saline is a hypotonic solution, and 3% saline is a hypertonic solution. Both of these IV solutions are inappropriate to co-administer with PRBCs. ✓ The nurse should remain with the client during a transfusion's first fifteen to thirty minutes to observe for a hemolytic or allergic reaction. ✓ The universal blood donor type is O negative; the universal blood type for recipients is AB positive. ✓ A unit of PRBCs should be transfused over 2-4 hours using Y-type tubing.

The nurse is gathering supplies for a transfusion of packed red blood cells (PRBCs). The nurse should obtain which intravenous (IV) fluid to accompany this transfusion? A. Lactated Ringers (LR) B. 0.9% saline C. 0.45% saline D. 3% saline

D Choice D is correct. The child's highest reading out of three times should be recorded (not the average). It is important that between each measurement, a 30-second rest is taken by the child. Choices A, B, and C are incorrect. The peak flow meter is a great tool for the client to determine the control of their asthma. Prior to the child measuring their peak flow, the device should be reset by sliding the marker (or arrow) on the meter by placing it at the bottom of the numbered scale. The child should not be sitting for this measurement; rather, they should be standing upright to allow for maximum chest expansion. The peak flow meter measures expiratory volume, so the child should be instructed to blow as hard and quickly as possible. Use of a Peak Expiratory Flow Meter 1. Before each use, make certain the sliding marker or arrow on the peak expiratory flow meter is at the bottom of the numbered scale. 2. Stand up straight. 3. Remove gum or food from your mouth. 4. Close your lips tightly around the mouthpiece. Be certain to keep your tongue away from the mouthpiece. 5. Blow out as hard and as quickly as you can, a "fast, hard puff." 6. Note the number by the marker on the numbered scale. 7. Repeat entire routine three times, but wait at least 30 seconds between each routine. 8. Record the highest of the three readings, not the average. 9. Measure your peak expiratory flow rate (PEFR) close to the same time and same way each day (e.g., morning and evening; before and 15 minutes after taking medication). 10. Keep a record of your PEFRs.

The nurse is instructing the parents of a child with asthma about a peak flow meter. Which statement, if made by the parents, would indicate effective teaching? A. "Before use, I should put the sliding marker at the top of the numbered scale." B. "I should have my child sit at a 45-degree angle while performing this procedure." C. "My child should inhale as quickly as they can through the mouthpiece." D. "I should record the highest of the three readings."

C Choice C is correct. Walkers should not be used on stairs or escalators. This places the client at significant risk of falling. Choices A, B, and D are incorrect. These observations reflect the effective use of a walker. The walker's height should be measured to the client's wrist crease, and the client should have elbow flexion of 15-30 degrees. As the client ambulates, they should lift the walker 6 to 10 inches and then advance the affected leg first. ADDITIONAL INFO ✓ The client with upper extremity weakness faces a challenge with walkers because they require the client to lift the device up and forward. ✓ Alternatively, walkers with wheels may be used; however, the walker can roll forward when weight is applied, causing the client to lose their balance. ✓ Walkers should never be used on stairs or an escalator.

The nurse is observing a client ambulate with a walker. It would require follow-up by the nurse if the client A. advances the walker 10 inches with each step. B. has their elbow flexed 15-30 degrees. C. ascends the stairs by advancing the walker first. D. advances the walker and then the affected leg.

C Choice C is correct. The new RN should not shake the vial but gently rotate it to ensure uniform suspension of insulin. Choice A is incorrect. Insulin injection sites should be rotated to prevent lipodystrophy. Asking the patient where the insulin was last injected gives the nurse an idea of where to insert the insulin next. Choice B is incorrect. The nurse should check the client's blood glucose levels before the administration of insulin to prevent hypoglycemia and to assess if the insulin dose needs to be adjusted. Choice D is incorrect. The insulin should be stored in a cool place away from direct sunlight. After it's opened; storing the insulin vial either in a refrigerator or at room temperature are both acceptable actions. Storing it in the fridge is recommended by the drug manufacturers. Injecting refrigerated insulin can be painful so patients may choose to store it at the room temperature after it is opened. Because both storing in the fridge or at room temperature are acceptable for the opened insulin vial, D is an appropriate nursing action and does not need further instructions.

The nurse is observing the newly registered nurse prepare and administer insulin to a patient. Which action by the new RN would necessitate further instructions from the supervising nurse? A. The new RN asks the client which site the insulin was last injected. B. The new RN checks the client's blood glucose levels prior to administering the insulin injection. C. The new RN shakes the insulin vial before withdrawing insulin. D. The new RN places the insulin in the medication fridge after drawing the needed amount of insulin.

B, C, D, E hoices B, C, D, and E are correct. Reducing medical errors is a continuous process,, and several proven factors may mitigate errors. These factors include: Having nurse-to-nurse bedside handoff reporting. Not only does this process increase client satisfaction by having the client participate, but it allows for errors to be noted by two individuals instead of one. For example, if an infusion pump is malfunctioning or administering the wrong dose. Bedside handoff reporting also mitigates distractions from being in the hallway or at the desk where it may be loud. A standardized handoff reporting (ISBAR) is helpful because it keeps communication concise to pertinent information. This standardized tool is useful because it keeps the communication process structured,, minimizing the risk of omission. Fatigue is a common source of errors. One way to minimize fatigue is to ensure all staff is taking uninterrupted breaks. Poor lighting and distractions continually contribute to errors, and increasing lighting around critical pieces of equipment, such as mediation dispensaries, may be helpful to reduce errors regarding reading product labels and drawing up accurate medication dosages. Choice A is incorrect. Verbal orders should be minimized because they allow for the skewing of the message through accents, dialects, etc. Written (or electronic) orders are highly preferred because of less interpretative error. Verbal orders should be limited to emergent situations. ADDITIONAL INFO ✓ Timely medication reconciliation is fundamental in the prevention of medication errors. ✓ Medication reconciliation is when the client's medications are inventoried and assessed for interactions, duplications, or omissions. ✓ Completing a medication reconciliation at admission, a transition of care, and discharge will identify potentially dangerous drug-to-drug interactions. ✓ Verbal and telephone errors are significant sources of medication errors. The nurse should limit these types of orders to emergent situations. These methods are discouraged because accents, dialects, and drug name pronunciations may skew the order being given. ✓ Documentation should be ongoing and not delayed. Vital details may be omitted from the documentation if it is intentionally delayed. ✓ Interruptions are a significant source of medication errors. Placing medication dispensing systems in quiet areas with adequate lighting is recommended.

The nurse is part of a committee tasked with reducing medical errors in the nursing unit. Which of the following recommendations should the nurse make to the committee? Select all that apply. A. Increase the number of verbal orders given from primary healthcare providers B. Nurse-to-nurse bedside handoff reporting C. Handoff reporting using the ISBAR framework D. Ensure staff are taking uninterrupted breaks E. Increase the lighting around the medication dispensing machines

B Choice B is correct. Patients with chronic kidney disease (CKD) retain electrolytes such as potassium, which may lead to imbalances. Hyperkalemia, or excess serum potassium levels, often results in cardiac dysrhythmias. Choice A is incorrect. While patients with chronic kidney disease may experience hypertension, a cardiac monitor does not evaluate the patient for this occurrence. Choice C is incorrect. Cardiac monitoring may show dysrhythmias, which could suggest the need for hemodialysis. However, this is not the primary method doctors use to evaluate this need. Choice D is incorrect. False episodes of asystole are not a concern with chronic kidney disease.

The nurse is placing the patient with chronic kidney disease on a cardiac monitor. This action is primarily performed because: A. Patients with chronic kidney disease are prone to hypertension B. Hyperkalemia may result in dysrhythmias C. Cardiac monitoring is necessary to evaluate the need for hemodialysis D. Patients with chronic kidney disease may experience false episodes of asystole

B Choice B is correct. The Human Papillomavirus (HPV) vaccine is the only vaccine proven to decrease the risk of cervical cancer. Nearly all cases of cervical cancer are linked to HPV and thus, the vaccine is an effective primary prevention method. Choices A, C, and D are incorrect. MMR vaccine is contraindicated during pregnancy. The client should not receive this vaccine or any other live vaccines during the pregnancy period. The seasonal influenza vaccine is an effective prevention method for children 6 months and older. Herpes zoster immunization is recommended starting at age 50. The MMR vaccine should not be administered to clients currently pregnant or four weeks prior to pregnancy. This is a two-series vaccine and is recommended for children starting at 12 to 15 months. The HPV Vaccine is available as a two or three-dose vaccine depending on which age it is started. Initial vaccination is recommended for males and females aged 11 to 12. The vaccine is recommended up to age 26. This vaccine is not recommended during pregnancy. The influenza vaccine comes in a variety of preparations - recombinant influenza vaccine, this vaccine is recommended for those aged 18 or older; inactivated influenza vaccine (IIV), which is recommended for ages six months or greater; live attenuated influenza vaccine (LAIV) that is given intranasally for those aged 2 through 49. The herpes zoster vaccine is recommended for individuals aged 50 or greater. This vaccine protects a client against shingles. This vaccine should be administered regardless of prior infection of varicella or herpes zoster.

The nurse is planning a staff development conference about vaccines. Which of the following information should the nurse include? A. MMR vaccine should be administered in the first trimester of pregnancy. B. Human Papillomavirus vaccine can reduce the risk of cervical cancer. C. Influenza vaccine may be administered to an infant at 3 months. D. Herpes zoster vaccine is recommended starting at age 40.

D Choice D is correct. An automatic external defibrillator (AED) should be requested immediately upon establishing that the client is in cardiac arrest. Waiting to request an AED could result in the delay of life-saving care. Choices A, B, and C are incorrect. An adult client should have their pulse palpated using the carotid artery. Chest recoil after every compression is essential to ensure optimal perfusion. Chest compressions for an adult should have a depth of two inches and be over the center breastbone. When performing CPR, the nurse needs to minimize interruptions and focus on providing effective compressions and ventilations ➢ A compression rate of 100-120/minute is desired ➢ An AED should be made available as urgently as possible ➢ Assessing for a pulse should not take more than ten seconds ➢ When obtaining a pulse for an infant, the nurse should assess the brachial artery. For a child and adult, the nurse will use the carotid artery ➢ Immediate family members should be allowed to be present during resuscitation as this has promoted better grieving

The nurse is precepting a graduate nurse as they perform resuscitation on an adult with cardiac arrest. Which action by the graduate requires immediate follow-up by the nurse? A. Assesses the client's pulse by palpating the carotid artery. B. Allows for chest recoil after every chest compression. C. Compresses at a depth of 2 inches on the center breastbone. D. Asks for an automatic external defibrillator after one cycle of CPR.

C Choice C is correct. A dressing is applied after the procedure, but a large pressure dressing is not required. Choice A is incorrect. The nurse needs to take pre-procedure vital signs, including weight, to establish a baseline. The pressure is taken before and after the procedure to indicate the effectiveness of the system in fluid removal. Choice B is incorrect. The client is made to void before the procedure to make sure that the bladder is not full and prevents it from being punctured. Choice D is incorrect. The client is maintained on bed rest after the procedure to assess the client for any complications.

The nurse is preparing a client for a paracentesis. Which nursing actions should not be included in the nursing care plan? A. Obtain the client's vital signs and weight before and after the procedure. B. Have the client void before the procedure. C. Apply a large pressure dressing after the procedure. D. Maintain the client on bed rest.

A Choice A is correct. When preparing to administer medications, the first nursing action is identifying the correct client. The other actions, such as verifying allergies, interactions, etc., cannot occur until the client's medical record is accessed, which starts with ascertaining the client's name and date of birth. Choices B, C, and D are incorrect. While verifying medication allergies, knowing what medications are ordered and their potential side effects is essential; the nurse's first action should be to verify that they have the right client via the two universal identifiers (name and date of birth). Vital signs are pertinent to the safe administration of certain medications (antihypertensive, etc.); however, the client's identity needs to be verified first.

The nurse is preparing to administer a prescribed medication to a client. The nurse should take which initial action? A. Verify the client's full name and date of birth B. Ask about any medication allergies C. Review the client's vital signs D. Review medications and potential interactions

B Choice B is correct. The desired outcome for a loop diuretic is the following - Reduction of blood pressure Reduction of pulmonary vascular resistance Reduction of systemic vascular resistance Reduction of central venous pressure Reduction of left ventricular end-diastolic pressure Choices A, C, and D are incorrect. It is not desired for a diuretic to increase CVP. CVP is the measurement of right ventricular end-diastolic pressure. Diuretics reduce blood volume, thereby reducing this pressure. The goal of a diuretic is to reduce the CVP. It is not desired for a client to have their GFR reduced. The goal for all clients is to have a high GFR, as this is an indicator of renal health. Prolonged exposure to loop diuretics may reduce the GFR, especially at aggressive doses. An increase in systemic vascular resistance is also not the desired effect of diuretics. SVR is the amount of force exerted on circulating blood by the body's vasculature. Less blood volume by the diuretic = a decrease in SVR. Loop diuretics act primarily along the thick ascending limb of the loop of Henle, blocking chloride and, secondarily, sodium resorption. Loop diuretics are also thought to activate renal prostaglandins, which dilate the blood vessels of the kidneys, the lungs, and the rest of the body (i.e., reduction in renal, pulmonary, and systemic vascular resistance). The hemodynamic effects of loop diuretics are a reduction in both the preload and central venous pressures (which are the filling pressures of the ventricles). These actions make them useful in treating the edema associated with heart failure, hepatic cirrhosis, and renal disease. Examples of loop diuretics include - bumetanide, ethacrynic acid, furosemide, and torsemide

The nurse is preparing to administer prescribed bumetanide to a client. Which clinical finding would indicate the desired outcome? A. Increase in central venous pressure B. Reduced cardiac preload and wall tension C. Decreased glomerular filtration rate D. Increase in systemic vascular resistance

C Choice C is correct. Since bladder muscles will not contract to increase intrabladder pressure and promote urination, the process is initiated manually. Overflow incontinence is continuous involuntary leakage or dribbling of urine that occurs with incomplete bladder emptying. It can be seen in men with an enlarged prostate and clients with a neurologic disorder (e.g. Parkinson's disease, spinal cord injury). An impaired neurologic function can interfere with the standard mechanisms of urine elimination, resulting in a neurogenic bladder. The client with a neurogenic bladder does not perceive bladder fullness and is unable to control the urinary sphincters. The bladder may become flaccid and distended or spastic, with frequent involuntary urination. Choices A, B, and D are incorrect. To promote continence, bladder contractions are required for habit training, bladder training, and increasing the tone of pelvic muscles.

Which focus is the nurse most likely to teach for a client with a flaccid bladder? A. Habit training: Attempt voiding at specific time periods. B. Bladder training: Delay voiding according to a pre-scheduled timetable. C. Credé's maneuver: Apply gentle manual pressure to the lower abdomen. D. Kegel exercises: Contract the pelvic muscles.

B Choice B is correct. The patient needs to be reinforced regarding a calcium-rich diet and calcium-rich foods. Milk and dairy products are some of the most common sources of dietary calcium. If the patient does not like milk or any other dairy product, the nurse should talk to him about different foods that are rich in calcium. Choice A is incorrect. Clients with osteomalacia need vitamin D to stimulate calcium absorption and mineralization. Vitamin D, exercise, and a calcium-rich diet are recommended. Walking under the sun stimulates vitamin D production in the body. Choice C is incorrect. Clients with osteomalacia need vitamin D to stimulate calcium absorption and mineralization. Vitamin D, exercise, and a calcium-rich diet are recommended. Enrolling in an exercise program indicates that the client understands the treatment regimen for osteomalacia. Choice D is incorrect. Clients with osteomalacia need vitamin D to stimulate calcium absorption and mineralization. Vitamin D, exercise, and a calcium-rich diet are recommended. Taking vitamin D supplements indicates that the client understands the treatment regimen for osteomalacia.

The nurse is talking to an elderly client with osteomalacia regarding ways to strengthen his bones. Which statement by the client would necessitate further teaching by the nurse? A. "I've started to walk more frequently under the sun." B. "I don't like dairy products so I've stopped eating them." C. "I've enrolled myself in an exercise program for seniors at the community center." D. "I've been taking Vitamin D supplements lately."

C Choice C is correct. Graves' disease is the most common cause of hyperthyroidism. When a client is taking antithyroid medication, such as methimazole, they should be taught about the warning signs of hypothyroidism (weight gain, constipation, anorexia). This could indicate that the dose needs to be decreased. Choices A, B, and D are incorrect. Constipation is a hallmark finding of hypothyroidism (Choice A). This would not be expected of Graves' disease. Thyroid replacement (Choice B) would be contraindicated for Graves' disease as this would worsen hyperthyroidism. Thyroid replacement should be taken first in the morning by patients with hypothyroidism, not hyperthyroidism. Fluid restrictions are not indicated for Graves' disease (Choice D). Graves' disease is an autoimmune disease and is the most common cause of hyperthyroidism. A hallmark finding of this disease is heat intolerance. Other manifestations of hyperthyroidism include exophthalmos, weight loss, irritability, and the thinning of scalp hair. The course of treatment is antithyroid medications or surgery.

The nurse is teaching a client who has Graves' disease about self-management. Which of the following should the nurse include in the teaching plan? A. Stool softeners can be taken daily to prevent constipation. B. Thyroid replacement should be taken first thing in the morning. C. Report any significant weight gain while taking the antithyroid medication. D. Maintain the prescribed fluid restriction to prevent fluid overload.

D Choice D is correct. Sudden onset of dysarthria and ataxia concerns for stroke. These manifestations require emergent prioritization because treatment is necessary to prevent further tissue damage. Choices A, B, and C are incorrect. Pleuritic chest pain would not be categorized as urgent because coughing indicates airway patency. Intermittent nausea and vomiting, without abdominal pain and cramping, would be triaged as nonurgent as this could be a normal part of pregnancy. An area of vesicles and the client reporting malaise would also be categorized as nonurgent. The emergent triage category implies that a condition exists that poses an immediate threat to life or limb. Conditions that should be triaged as emergent include: active hemorrhage, unstable vital signs, significant trauma, chest pain, and manifestations of a stroke. The urgent triage category indicates that the client should be treated quickly but that an immediate threat to life does not exist at the moment. Conditions that typically fall into the urgent category are those with a new onset of pneumonia (as long as respiratory failure does not appear imminent), renal colic, abdominal pain, complex lacerations not associated with major hemorrhage, displaced fractures or dislocations, and temperature higher than 101°F (38.3°C). Those triaged as non-urgent can generally tolerate waiting several hours for health care services without a significant risk for clinical deterioration. Conditions within this classification include clients with sprains and strains, simple fractures, general skin rashes, and uncomplicated urinary tract infections.

The nurse is triaging clients in the emergency department (ED). Which client should the nurse triage as emergent? A client A. reporting pleuritic chest pain with a productive cough. B. who is pregnant and reporting intermittent nausea and vomiting. C. who has an isolated area of reddened vesicles and malaise. D. with sudden onset of ataxia and dysarthria.

B Choice B is correct. Trichomoniasis patients would yield a malodorous, thin, yellow discharge. Trichomoniasis is caused by a protozoon, Trichomonas vaginalis. Choice A is incorrect. A white, "cheesy" discharge is indicative of moniliasis or candidiasis, which is caused by Candida albicans. Choice C is incorrect. Grayish-white, malodorous discharges would indicate bacterial vaginosis. Choice D is incorrect. Patients with trichomoniasis yield a malodorous, thin, yellow discharge.

The nurse is working at a women's health clinic. A patient comes in suspected of having trichomoniasis. Upon physical examination of the perineal region, the nurse should expect which type of sign? A. White, "cheesy" discharge B. Malodorous, thin, yellow discharge C. Grayish-white, malodorous discharge D. No vaginal discharge

C Choice C is correct. This statement correctly describes cystic fibrosis as an inherited disease causing excessive, thick mucus to build up in the body and cause blockages. Choice A is incorrect. Cystic fibrosis is NOT an inherited disease that causes inflammation and hypersensitivity of the airway. A disease that causes inflammation and hypersensitivity of the airway is asthma, not CF. Choice B is incorrect. Cystic fibrosis is NOT an infectious disease that causes inflammation and fluid accumulation in the alveoli of the lungs. Pneumonia is an infectious disease that causes inflammation and fluid accumulation in the alveoli of the lungs. Choice D is incorrect. Cystic fibrosis is NOT an acquired disease that causes inflammation and swelling of the epiglottis. Epiglottitis is a disease that causes inflammation and swelling of the epiglottis.

The nurse is working with an advocacy group to raise awareness about cystic fibrosis. Which statement best explains the condition? A. "It is an inherited disease that causes inflammation and hypersensitivity of the airway." B. "It is an infectious disease causing inflammation and fluid accumulation in the alveoli of the lungs." C. "It is an inherited disease causing excessive, thick mucus to build up in the body and cause blockages." D. "It is an acquired disease that causes inflammation and swelling of the epiglottis."

B, C, D B is correct. The patient with AML will show a decreased neutrophil count. The question tells you that the patient is neutropenic, meaning that they have a lower than an average number of neutrophils. Due to the proliferation of blast cells, healthy, mature neutrophils are decreased. This leads to a reduced ability for the body to fight infection. C is correct. The patient with AML will have an increased percentage of blast cells. Blast cells are immature neutrophils. There is a proliferation of these immature neutrophils, or blasts, in the bone marrow in patients with AML. This is what causes other healthy cells to die and leads to many signs and symptoms. D is correct. The patient with AML will have a decreased platelet count. This is because of the proliferation of blast cells and suppression of other standard and healthy cells in her bone marrow. The decrease in platelets often can lead to longer clotting times, increased bleeding, and increased bruising in AML. Choice A is incorrect. The patient with AML will likely experience a decrease in erythrocytes or red blood cells. This is because of the proliferation of blast cells and suppression of other standard and healthy cells in her bone marrow. The decrease in erythrocytes often leads to anemia as part of the AML.

The nurse knows that her patient with acute myeloid leukemia (AML) is neutropenic. When reviewing their CBC results in the morning, which of the following does she expect? Select all that apply. A. Increased erythrocyte count B. Decreased neutrophil count C. Increased blast percentage D. Decreased platelet count

A Choice A is correct. Open suction of a tracheostomy tube requires an aseptic technique. After setting up a sterile field and applying sterile gloves, the nurse would designate one hand as contaminated and ensure the other remains sterile. The contaminated hand should be used to connect/disconnect the catheter tubing, use the resuscitation bag, and operate the suction control. If preoxygenation is indicated, the nurse would use the contaminated hand to administer it. Choice B is incorrect. The sterile hand would be the correct choice for advancing the catheter, but suction should never be applied during insertion. Intermittent suction would only be used while withdrawing the catheter. Choice C is incorrect. The nurse has performed the procedure steps correctly so far and has not taken any action that would compromise the sterile field or require re-starting the procedure. Choice D is incorrect. The nurse should assess the patient's oxygenation status prior to setting up the sterile field and starting the procedure to use as a baseline for monitoring the patient's response to the procedure.

The nurse prepares to suction a tracheostomy tube to help clear a patient's secretions. After opening the package, filling the cup with sterile water, and putting on sterile gloves, the nurse uses one hand to connect the catheter to the suction. What action would be most appropriate for the nurse to take next? A. Use the contaminated hand to preoxygenate the patient prior to suction. B. Use the sterile hand to slowly insert the catheter while applying intermittent suction. C. Restart the procedure due to contamination after applying sterile gloves. D. Assess the patient's baseline oxygenation status.

A Choice A is correct. This prescription is inaccurate and requires clarification with the PHCP before moving forward. This medication was prescribed as a volume of 5 mL, not the precise dosage amount to be administered (for example, it is okay to be prescribed 5 mg of metoprolol, not 5 mL). The nurse needs an accurate prescription that is complete before executing other steps in the medication administration process. Choices B, C, and D are incorrect. These actions are all correct to perform if the nurse has an accurate prescription. The prescription of metoprolol is incomplete because the actual dose was not provided. Metoprolol is a beta-blocker indicated in treating hypertension and specific dysrhythmias such as atrial fibrillation. ADDITIONAL INFO The nurse must clarify any inaccurate or incomplete prescriptions to maintain client safety. Components of a medication order include the name of the drug, dosage, route, and frequency.

The nurse receives a prescription from the primary healthcare provider (PHCP) for metoprolol 5 mL intravenous (IV) push x 1 dose. The nurse should take which priority action before administering the medication? A. Clarify the prescription with the primary healthcare provider (PHCP) B. Assess vital signs C. Obtain a 5 mL syringe D. Assess the client's allergies

A Choice A is correct. Albuterol is a bronchodilator that is used for asthma exacerbations. Adversely, this medication may lower serum potassium levels. The nurse should question this order as this medication may decrease the potassium further. Choices B, C, and D are incorrect. Clozapine is an atypical antipsychotic used in the treatment of severe schizophrenia. Lisinopril is an ACE inhibitor indicated for heart failure and hypertension. Verapamil is commonly used as prophylaxis for migraine headaches; this medication may also be used for individuals with hypertension. Albuterol is a short-acting bronchodilator. Common side effects of albuterol include tremors, tachycardia, palpitations, and metabolic disturbances such as hypokalemia and hyperglycemia. This medication is emergently indicated for asthma exacerbations. Sometimes, it is used as an adjunctive agent for treating hyperkalemia because it can drive potassium into the cells and lower serum potassium.

The nurse reviews prescriptions for assigned clients. Which prescription should the nurse question? A. Albuterol via nebulizer for a client with hypokalemia. B. Clozapine for a client with severe schizophrenia. C. Lisinopril for a client with congestive heart failure. D. Verapamil for a client with migraine headaches.

B Choice B is correct. The most common side effects of rivastigmine are flu-like symptoms, dizziness, and weight loss. The FDA has approved limited drugs for Alzheimer's Disease. The most effective medications act by intensifying the effect of acetylcholine at the cholinergic receptor. Acetylcholine is naturally degraded in the synapse by the enzyme acetylcholinesterase. When acetylcholinesterase is inhibited, acetylcholine levels increase and significantly affect the receptors. Choice A is incorrect. Liver toxicity is not an anticipated side effect of rivastigmine. Choice C is incorrect. Renal failure is not an anticipated side effect of rivastigmine. Choice D is incorrect. Extrapyramidal symptoms are not an anticipated side effect of rivastigmine.

The nurse should assess an Alzheimer's patient who has been started on rivastigmine for which of the following side effects? A. Liver toxicity B. Weight loss C. Renal failure D. Extrapyramidal side effects

D Choice D is correct. Unfamiliar surroundings is a significant risk factor for falls, especially in the elderly. The hospitalized patient may become confused or bump into furniture, which could result in a fall. Age-related changes may affect the mobility and safety of older adults. For example, decreased muscle strength, reduced balance, and osteoporosis put older adults at risk for falls and fractures. For health promotion, the nurse assesses the musculoskeletal functioning of the older adult and identifies any risk factors that may contribute to falls or the ability of the older adult to perform ADLs. Health promotion interventions often include providing information about the risk factors for osteoporosis and the importance of adequate intake of calcium and vitamin D. Choice A is incorrect. An individual's source of income has no bearing on the risk of falls. Choice B is incorrect. While age-related changes may cause weakness and a slowed reflex response, age is not the most likely risk factor for falls among the available answer choices. Choice C is incorrect. An elderly client living in his own home will be less likely to fall than a client who is in unfamiliar surroundings.

The nurse working with geriatric clients understands that falls are likely to occur in elderly patients who are: A. Living on disability insurance B. In their 80s C. Living in their own home D. Hospitalized

B Choice B is correct. Induction of vomiting when a victim has ingested hydrocarbons is contraindicated. Vomiting may lead to inhalation of the poison, worsening the situation. Choice A is incorrect. This is a correct statement. Proper labeling can help prevent accidental ingestion of poisons at home. Choice C is incorrect. This is a correct statement. Diluting the poison can buy some time in getting the child/victim some needed help. Choice D is incorrect. This is a correct statement. Poisonous materials should always be stored away from children and must be locked.

The school nurse is talking to a group of mothers regarding poison prevention and management. Which statement by the mothers indicates a need for further teaching? A. "I need to properly label the containers of poisonous liquids." B. "I need to make my child vomit in the instance he ingests gasoline." C. "I can give my child milk or some water to dilute the poison while I rush him to the hospital." D. "All poisonous materials should be stored away from children."

C Choice C is correct. You should respond to this client's statement with, "Hyperalimentation is one kind of parenteral nutrition that gives you feedings with a special IV line." Parenteral nutrition, which is synonymous with hyperalimentation and IV hyperalimentation, provides the client with complete food when it is indicated for a client such as one who is adversely affected Choice A is incorrect. You should not respond to this client's statement with, "Your doctor is thinking about total parenteral nutrition and not hyperalimentation" because parenteral nutrition is hyperalimentation; parenteral nutrition is synonymous with hyperalimentation and IV hyperalimentation. Choice B is incorrect. You should not respond to this client's statement with, "Hyperalimentation is one kind of enteral nutrition that gives you feedings with a tube" because hyperalimentation is parenteral nutrition and not enteral nutrition. Choice D is incorrect. You should not respond to this client's statement with, "You should choose to have enteral nutrition and not accept hyperalimentation" because this is coercive and contrary to the client's right to make an informed decision about any or all care that is being considered.

Total parenteral nutrition (TPN) is being considered for your client. Your client tells you, "My doctor is thinking about hyperalimentation, and I know nothing about it. Can you tell me what it is?". You should respond to this client's statement with: A. "Your doctor is thinking about total parenteral nutrition and not hyperalimentation." B. "Hyperalimentation is one kind of enteral nutrition that gives you feedings with a tube." C. "Hyperalimentation is one kind of parenteral nutrition that gives you feedings with a special IV line." D. "You should choose to have enteral nutrition and not accept hyperalimentation."

C Choice C is correct. Studies of restraint-related deaths have shown that people of small stature are more likely to slip through or between the side rails. Choice A is incorrect. The desire to prevent a patient from wandering is not sufficient reason for the use of side rails. Choice B is incorrect. A history of falls from a bed with raised side rails carries a significant risk for a future serious incident. Choice D is incorrect. Creative use of alternative measures indicates respect for the patient's dignity and may, in fact, prevent more serious fall-related injuries.

What consideration should the nurse keep in mind regarding the use of side rails for a confused patient? A. They prevent confused patients from wandering. B. A history of a previous fall from a bed with raised side rails is insignificant. C. A person of small stature is at increased risk for injury from entrapment. D. Alternative measures are ineffective to prevent wandering.

B Choice B is correct. Circadian rhythm is defined as our 24-hour biological clock that is primarily one that functions best with daytime wakefulness/activity and nighttime sleep. When clients are in synchrony with their biological clock, humans function optimally because many of our essential rational physiological and mental functions like blood pressure, body temperature, and levels of alertness/performance are at their optimal levels. Choice A is incorrect. REM sleep is a phase of the sleep cycle and not the term that is used to describe a human's innate biological clock relating to daytime and nighttime wakefulness/activity. Choice C is incorrect. Diurnal activity is daytime activity and not the term that is used to describe a human's innate biological clock relating to daytime and nighttime wakefulness/activity. Choice D is incorrect. Nocturnal activity is nighttime activity and not the term that is used to describe a human's innate biological clock relating to daytime and nighttime wakefulness/activity.

What is the term that is used to describe a human's innate biological clock relating to daytime and nighttime wakefulness and activity? A. REM sleep B. Circadian rhythm C. Diurnal activity D. Nocturnal activity

D Choice D is correct. Bruising or bleeding when taking antivirals could indicate possible bone marrow suppression and may require dosage adjustments or a medication change. Choice A is incorrect. While abuse of any patient is a possibility, the question is regarding the use of antivirals and the symptom of bruising, which is an indication of bone marrow suppression. Choice B is incorrect. Bruising is not an adverse reaction, but may be an indication of something more serious. Choice C is incorrect. While the patient may not be taking the medication correctly, the most appropriate answer is choice D. The nurse should ask the patient to clarify how much medicine he/she is taking and how often. However, the most likely source of bruising (among these answers) is the suppression of bone marrow.

When observing a patient on antivirals. The nurse notices the patient has developed bruising. This could indicate which of the following? A. The patient is being abused by a family member. B. The patient is experiencing minor adverse reactions C. The patient is not taking the medications as ordered. D. The patient may be experiencing bone marrow suppression.

B Choice B is correct. Cognitive-behavioral therapy (CBT) is a treatment that combines cognitive psychotherapy and behavioral psychotherapy. CBT is recommended as a first-line treatment approach for specific phobias. The behavioral component of CBT involves repeated exposure to the feared situations and thereby promotes fear reduction. For example, a virtual reality exposure strategy using computer technology can be used to simulate real-life situations (e.g. treating fear of flying with repeated exposure in a flight simulator). The cognitive component of CBT facilitates the client to identify the maladaptive thoughts relating to stressors and replace them with realistic thoughts. For example, a client with a specific phobia of elevators may incorrectly believe there is an extremely high chance of getting stuck while in an elevator. This distorted belief can be addressed with cognitive therapy and thereby reduce fear. CBT is also used for generalized anxiety disorder, panic disorder, eating disorders (anorexia nervosa, bulimia), and obsessive-compulsive disorder (OCD). Choice A is incorrect. Behavioral psychotherapy is useful for patients adversely affected by substance-related disorders and other addictive disorders. Some of the techniques used with behavioral therapy include operant conditioning as put forth by Skinner, aversion therapy, desensitization therapy, modeling, and complementary and alternative stress management techniques. Choice C is incorrect. Psychoanalysis deals with the client's subconscious and focuses on the past and current issues. Psychoanalysis and psychodynamic psychotherapy have been used in treating anti-social personality disorders. Choice D is incorrect. Cognitive psychotherapy is primarily used to treat patients with depression, anxiety disorders, or eating disorders. Cognitive therapy is aimed at altering the client's perspective and attitudes relating to stressors.

Which form of therapy would most likely be used to treat a group of clients affected by phobias? A. Behavioral psychotherapy B. Cognitive behavioral psychotherapy C. Psychoanalysis D. Cognitive psychotherapy

C, D Choices C and D are correct. Since the purposes of these are for the benefit of the patient and the information is shared with health care providers directly involved in the patient's care, these are acceptable under HIPAA. The HIPAA Privacy Rules require healthcare agencies to make reasonable efforts to limit the use of, disclosure of, and requests for protected health information to the minimum necessary to accomplish the intended purpose. Choice A is incorrect. Talking with a colleague is acceptable in the context of learning, such as a post-conference. However, elevators, cafeterias, and other public spaces are inappropriate locations because such actions violate the patient's right to privacy. Additionally, visitors and other patients may become anxious or fearful when overhearing details related to illness, procedures, and other health-related concerns. Choice B is incorrect. Consulting with an instructor is appropriate, but it should be done in designated areas that promote privacy and confidentiality of patient information (not in public places and hallways).

Which of the following are acceptable under the HIPAA Privacy Rule? Select all that apply. A. Describe the patient assessment findings to a colleague during a business lunch. B. Consult in the hallway with an instructor about the patient's abnormal findings. C. Communicate report with the next nurse during shift change. D. Communicate with the primary care provider about a patient's change in assessment.

B Choice B is correct. The elderly experience increased autoimmune responses. This increases the risk of diseases such as rheumatoid arthritis and other collagen disorders. Choice A is incorrect. Lymphoid tissue is decreased in the elderly, which results in lowered immune responses. Choice C is incorrect. The number of circulating lymphocytes reduces by nearly 15 percent. In addition to a decline in antibody-antigen reaction, this makes the older client more susceptible to infection. Choice D is incorrect. The number of T and B cells produced by the older client is decreased.

Which of the following are clinical manifestations of the aging immune system and should alert the nurse of increased susceptibility to illness? A. Increased lymph tissue B. Increased autoimmune responses C. Increased circulation of lymphocytes D. Increased T and B cell production

C Choice C is correct. Performing range of motion exercises will help decrease the risk of further atrophy and should be encouraged. Huntington's disease is a progressive condition that can lead to muscle atrophy and potential contractures. The patient in this situation should be given a program of range of motion exercises in which he may need assistance. The nurse can help the patient to increase his range of motion and to prevent worsening of contractures by improving flexibility and reducing rigidity. Choice A is incorrect. Contractures and lack of mobility put a client at risk of compromised skin integrity. However, the nurse is responsible for monitoring for any signs of breakdown. Choice B is incorrect. The client has developed contractures and muscle atrophy. He may be unable to reposition himself without assistance. Choice D is incorrect. The client may not verbalize the effects on ADLs.

Which of the following best describes an appropriate outcome for a 75-yr-old patient with a history of Huntington's disease, which has developed contractures? A. The patient will monitor for signs of skin breakdown as a result of the contractures. B. The patient will learn to reposition himself in bed and in his chair without assistance. C. The patient will participate in range of motion exercises to reduce the effects of contractures. D. The patient will verbalize the effects of contractures on activities of daily living.

B, C Choices B and C are correct. Parathyroid hormone (PTH) causes increased phosphate to be lost in the urine. Therefore, a high serum phosphate would cause the release of parathyroid hormone to help the body decrease serum phosphate (Choice B). Catecholamines, such as epinephrine, have a direct effect on parathyroid hormone. Increased catecholamines cause an increase in the secretion of parathyroid hormone (Choice C). Choice A is incorrect High serum magnesium would cause decrease of parathyroid hormone. Usually, parathyroid hormone increases the reabsorption of magnesium by the renal tubules, therefore, increasing serum magnesium. When serum magnesium gets high, the body suppresses parathyroid hormone ( feedback inhibition) to help reduce the magnesium level. Similarly, Low levels of magnesium signal the parathyroid gland to secrete PTH, which signals the kidneys to retain magnesium, raising magnesium levels in the body. Choice D is incorrect. Parathyroid hormone causes accelerated bone breakdown, which thereby releases calcium into the blood. Low serum calcium would stimulate the release of parathyroid hormone to help the body increase serum calcium. On the other hand, high serum calcium inhibits the PTH release.

Which of the following conditions result in the release of parathyroid hormone (PTH)? Select all that apply. A. High serum magnesium B. High serum phosphate C. Catecholamine release D. High serum calcium

A Choice A is correct. The upper outer quadrant is the site of most breast tumors. In the upper outer quadrant, the nurse should examine the axillary's tail of Spence, the cone-shaped breast tissue that projects up into the axilla, close to the intimate group of axillary lymph nodes. Choices B, C, and D are incorrect. Although the information in these answer choices is correct, they do not reflect the essential reason for examining the upper outer quadrant of the breast during the assessment.

Why is it important to examine the upper outer quadrant of the breast when performing a breast assessment? A. This is where most breast tumors develop. B. This part of the breast is more prone to injury and calcifications. C. This is the largest quadrant of the breast. D. This is where most of the suspensory ligaments attach.

B, D Choices B and D are correct. Orange juice is high in vitamin C, which will help increase the absorption of iron. Also, this will make taking the supplement easier on the stomach and many say it helps with the bad taste (Choice B). If the healthcare provider orders an oral suspension iron supplementation, you should teach your patient to drink it through a straw to avoid staining their teeth. Alternatively, if you are administering the medication to a young child who cannot drink through a straw, you can pull it up in a syringe and squirt it into the back of their mouth behind their teeth (Choice D). Choice A is incorrect. Taking an iron supplement on a full stomach will not allow for proper absorption. You must educate the patient to take their iron supplement on an empty stomach. Choice C is incorrect. Black stools are an expected side effect of iron supplementation. Patients do not need to report black stools to their doctor if they are taking an iron supplement. The nurse should warn them to expect this side effect so that they are not alarmed.

Which of the following educational points are correct when teaching a patient about iron supplementation? Select all that apply. A. Take the iron supplement 30 minutes after a meal. B. Drink a glass of orange juice with your iron supplement. C. Report any black stools to your doctor. D. Drink the iron suspension with a straw.

C Choice C is correct. Immediately calming the infant is the nursing priority during a tet spell. While the infant is crying, their pulmonary vascular resistance is increasing leading to decreased oxygenated blood and more cyanosis. By calming them down you will immediately be decreasing their pulmonary vascular resistance so that blood can flow to the lungs and provide oxygen to the body. This is the first action that the nurse should take. Choice A is incorrect. While propranolol may be used in children with tetralogy of Fallot, it will not be the priority nursing action for the infant experiencing a tet spell. It will be given much later if necessary. Choice B is incorrect. Sodium bicarbonate may be needed at some point during a tet spell if it is not resolving, but would not be indicated as soon as it starts and would not be the priority nursing action. Choice D is incorrect. While the nurse will need to notify the healthcare provider of the spell and may need additional assistance, this still isn't the priority action. There is another action listed that will immediately help the infant and should be the priority.

Which of the following is the nurse's priority nursing action for the infant experiencing a tetralogy of Fallot (tet) spell? A. Administer propranolol B. Administer sodium bicarbonate C. Calm the infant D. Notify the healthcare provider

B Choice B is correct. Lipid profile helps physicians determine the patient's risk of developing heart disease. It is recommended that individuals have a lipid profile done at least every five years as part of a regular medical exam. 6.5 mmol/L exceeds the "high normal" total cholesterol level. The average total cholesterol level is 3.5 to 5.0 mmol/L. In milligrams, total cholesterol of 200 milligrams per deciliter (mg/dL) or less is considered desirable for adults Choices A, C, and D are incorrect. The normal lipid levels for these tests include: Triglycerides: 50-150 mg/dL High-density lipoprotein (HDL): 40-80 mg/dL Low-density lipoprotein (LDL): 85-125 mg/dL

Which of the following lipid levels are out of range and should be reported to the physician? A. Triglycerides: 75 mg/dL B. Total cholesterol: 6.5 mmol/L C. High-density lipoprotein (HDL): 60 mg/dL D. Low-density Lipoprotein (LDL): 95 mg/dL

A, B Choices A and B are correct. The vasodilation properties of a beta-blocker mean that they decrease blood pressure. This is because the beta-blockers are blocking the receptor sites for your catecholamine, so they cannot do their job and cause vasoconstriction (Choice A). Beta-blockers decrease the workload of the heart. This is because of the vasodilation, subsequent decrease in blood pressure, and then fall in afterload. Remember, afterload is the pressure against which the left ventricle must pump. With decreased blood pressure, we reduce afterload. With reduced afterload, the left ventricle does not have to work as hard to pump blood to the body. Therefore, beta-blockers decrease the workload of the heart (Choice B). Beta-blockers block the beta cells of the body. Beta cells are receptor sites for catecholamines, such as epinephrine and norepinephrine. When we block the receptor sites for the catecholamines, they cannot do their job. Catecholamines function to increase everything - increase blood pressure, increase pulse, increase contractility, and cause vasoconstriction. This is because they are fight-or-flight hormones. They get the body excited and ready to go. So, when beta-blockers block them, everything decreases. Your body vasodilates, the heart slows down, and the blood pressure decreases. Choice C is incorrect. Beta-blockers decrease contractility, not increase; this is because they are blocking those beta cell receptor sites for catecholamines such as epinephrine and norepinephrine. The catecholamines work to increase contractility, but they are blocked by the beta-blockers. So, beta-blockers decrease contractility. Choice D is incorrect. Beta-blockers decrease cardiac output, not increase; this is because of the decreased contractility. While the catecholamine receptor sites are blocked, they are unable to cause increased contractility and the contractility of the heart decreases. With decreased contractility comes a decreased stroke volume. Since CO = HR x SV, a reduced stroke volume means a reduced cardiac output.

Which of the following statements are true regarding beta blockers' mechanism of action? Select all that apply. A. Decrease blood pressure B. Decrease workload of the heart C. Increase contractility D. Increase cardiac output

A, C, F Choices A, C, and F are correct. A: Sedatives, such as diazepam (Valium), midazolam (Versed), and lorazepam (Ativan), are given to alleviate anxiety and decrease the recall of events related to surgery. C: Anticholinergics, such as atropine and glycopyrrolate (Robinul), are given to decrease pulmonary and oral secretions in order to prevent laryngospasm. F: Neuroleptanalgesic agents, such as fentanyl citrate-droperidol (Innovar), are given to cause a general state of calm and sleepiness. Choices B, D, and E are incorrect. B and E: Histamine-2 receptor blockers, such as cimetidine (Tagamet) and ranitidine (Zantac) are given to decrease gastric acidity and volume. D: Narcotic analgesics, such as morphine, are given to decrease the amount of anesthetic agent needed.

Which statements describe the action of the medications? Select all that apply. A. Diazepam is given to alleviate anxiety. B. Ranitidine is given to facilitate patient sedation. C. Atropine is given to decrease oral secretions. D. Morphine is given to depress respiratory function. E. Cimetidine is given to prevent laryngospasm. F. Fentanyl citrate-droperidol is given to facilitate a sense of calm.

A Choice A is correct. Upon finding a fire, the nurse's first step should be to evacuate any patients who are in immediate danger. Once this step is completed, she must activate the fire alarm. Choice B is incorrect. The nurse doesn't need to focus on extinguishing the fire until the fire alarm has been activated and the fire confined. Choice C is incorrect. Contacting the nursing supervisor is not a priority during a fire-related emergency. Choice D is incorrect. Confining the fire is not necessary until the fire alarm has been activated.

While performing in-room charting on a patient. The nurse smells smoke from the bathroom upon entering the lavatory. She finds a trashcan on fire. After evacuating the patients from the room. What is her priority action? A. Activate the fire alarm. B. Extinguish the fire. C. Contact the nursing supervisor. D. Close the door to the bathroom.

A Choice A is correct. Asthma typically begins in early life, whereas symptoms of CHF and COPD usually develop later in life. Bronchiectasis typically presents with signs and symptoms of a recent infection, including large amounts of bronchial secretions. Asthma symptoms tend to come and go with symptoms being worse at night. There is often a family history of asthma, and it usually occurs in obese patients.

You are caring for a newly admitted obese patient in the ICU. The patient has a history of smoking. She states that her symptoms started early in life and are worse at night. She denies any history of recent fever or chills. You notice wheezing and stridor upon assessment. You expect the diagnosis for this patient will be: A. Asthma B. Bronchiectasis C. Congestive heart failure (CHF) D. Chronic obstructive pulmonary disease (COPD)

A Choice A is correct. The need that you should include in a class related to nutrition during pregnancy for a group of young pregnant women is the need to increase caloric intake by about 340 calories during the second trimester of gestation. This caloric increase is necessary to support the growth and development of the fetus. Similarly, a caloric increase of about 450 calories is indicated during the third trimester of gestation for the same reason. Choice B is incorrect. Although there is a need to increase caloric intake during the second trimester of gestation, this increase is less than 370 calories. Choice C is incorrect. Although there is a need to increase caloric intake during the third trimester of gestation, this increase is more than 340 calories. Choice D is incorrect. Although there is a need to increase caloric intake during the third trimester of gestation, this increase is more than 370 calories.

You are planning an educational series of classes for young pregnant women. Which of the following needs should you include in a class related to nutrition during pregnancy? A. The need to increase caloric intake by about 340 calories during the second trimester of gestation. B. The need to increase caloric intake by about 370 calories during the second trimester of gestation. C. The need to increase caloric intake by about 340 calories during the third trimester of gestation. D. The need to increase caloric intake by about 370 calories during the third trimester of gestation.

B Choice B is correct. The nurse must directly monitor the client and their responses to the transfusion continuously for at least 15 minutes after the bleeding began. This is because transfusion reactions are most likely to manifest within 15 minutes after the transfusion starts. Choice A is incorrect. Although you would directly monitor the client and their responses to the transfusion continuously for some time, this duration is typically less than 30 minutes after the bleeding began. Choice C is incorrect. You would start the infusion by adjusting the rate of the injection to a volume that is less than 30 mL per minute; rates of 30 mL per minute can lead to a more severe reaction than a much lower rate would. Choice D is incorrect. You would start the infusion by adjusting the rate of the injection to a volume that is less than 20 mL per minute; rates of 20 mL per minute can lead to a more severe reaction than a much lower rate would.

You are ready to administer a unit of packed red blood cells to your client. Which of the following nursing interventions is a high priority for you once the transfusion has begun? A. Directly monitor the client and their responses to the transfusion continuously for at least 30 minutes after the transfusion began. B. Directly monitor the client and their responses to the transfusion continuously for at least 15 minutes after the transfusion began. C. Start the infusion by adjusting the rate of the infusion to less than 30 mL per minute. D. Start the infusion by adjusting the rate of the infusion to less than 20 mL per minute.

A, C, D Choices A, C, and D are correct. A is correct. Hydration is an essential component of preventing a sickle cell crisis, so this is a critical education. By drinking lots of water, the boy will increase the volume in his vascular space with fluid, substantially "thinning out" the sickled cells. In other words, they will not be as concentrated anymore. This will help to prevent the sickled cavities from snagging on vessels, creating occlusions, and causing a crisis. C is correct. If patients with sickle cell disease fly on airplanes, they need to take proper precautions. In planes, you are at a very high altitude where there is much less oxygen. This can be a trigger for a sickle cell crisis because it leads to a high oxygen demand state. Precautions to take if they do so include staying very well hydrated, and possibly having an oxygen mask available. Patients with many crisis events may need to avoid flying on airplanes. D is correct. The parents need to know to call the child's primary care doctor if he is ill with a fever. Since the body demands more oxygen when it is febrile, temperatures are a trigger for sickle cell crises and must be treated promptly. Choice B is incorrect. While promoting a healthy lifestyle is always essential, vigorous exercise is a specific trigger for a sickle cell crisis. During strenuous exercise, the tissues have a high oxygen demand and the sickled cells are unable to deliver a sufficient amount of O2, which results in a crisis. So for this patient, 60 minutes of vigorous exercise every day would not be a good recommendation.

You are reinforcing the education provided to your 8-year-old patient diagnosed with sickle cell anemia. He had three sickle crises events this year. Which of the following points do you enforce with him and his parents to help prevent more sickle cell crises? Select all that apply. A. Drink lots of water B. Perform vigorous exercise for 60 minutes a day C. Take proper precautions if flying on airplanes D. Call the PCP if he becomes febrile

B Choice B is correct. Lovastatin increases the effects of warfarin, so the nurse should incorporate this knowledge related to an increased influence of the anticoagulant, warfarin, into the plan of care. Choice A is incorrect. Lovastatin increases the effects of warfarin. Choice C is incorrect. Lovastatin has known effects on warfarin, an anticoagulant medication, so the nurse should consider this when planning care. Choice D is incorrect. Combining lovastatin and warfarin does not cause respiratory depression.

You are taking care of a client who is taking warfarin and lovastatin. Which statement about the interaction warfarin and lovastatin should you incorporate into your plan of care? A. Lovastatin decreases the effects of the warfarin. B. Lovastatin increases the effects of the warfarin. C. Lovastatin has no known effects on the warfarin. D. Combining lovastatin and warfarin causes respiratory depression.

B Choice B is correct. The theory of aging that supports your belief that strict infection control prevention measures are necessary is the 'Immunological Theory of Aging'. The 'Immunological Theory of Aging' states that aging leads to the decline of the person's defensive immune system and the decreased ability of the antibodies to protect us against infection. Programmed theories assert that the human body is designed to age and there is a certain biological timeline that bodies follow. All of these theories share the idea that aging is natural and "programmed" into the body. Error theories, such as the 'Rate of Living Theory', assert that aging is caused by environmental damage to the body's systems, which accumulates over time. Choices A, C, and D are incorrect. The 'Programmed Longevity theory of aging' states that genetic instability and changes occur such as some genes turning on and off lead to the aging process; the 'Endocrine Theory of aging' states that aging results from hormonal changes and the biological clock's ticking; and 'Rate of Living Theory' states that one's longevity is the result of one's rate of oxygen basal metabolism.

You are the nursing supervisor in a long-term care facility. One of the major considerations that you apply into your practice is strict infection control prevention measures because you are knowledgeable about the fact that the normal aging process is associated with the deterioration of the body's normal defenses. Which theory of aging supports your belief that strict infection control prevention measures are necessary? A. The Programmed Longevity Theory B. The Immunological Theory of Aging C. The Endocrine Theory D. The Rate of Living Theory

B Choice B is correct. The FLACC Pain Scale is a valid and reliable pain assessment tool for the assessment of pain/pain intensity among clients who range in age from 2 years old to 10 years old. The FLACC scale consists of pain behavior assessments, such as: F: Face expressions, such as grimacing L: Movement of the legs A: Level of activity C: Crying C: Degree to which the child is consolable Choice A is incorrect. You would not use the Neonatal Infant Pain Scale (NIPS). The NIPS scale is only valid and reliable for infants and neonates. Choice C is incorrect. You would not use the PAINAD Pain Scale. The PAINAD Pain Scale is only valid and reliable for elderly adults who are adversely affected with advanced dementia. Choice D is incorrect. You would not use the CRIES pain scale because this pain assessment tool is valid and reliable only for neonates.

You are working in a pediatric unit with acutely and chronically ill pediatric clients. Which of the following pain assessment tools would you most likely use to assess pain and pain intensity among your clients who range in age from 2-years-old to 10-years-old? A. NIPS Pain Scale B. FLACC Scale C. PAINAD Pain Scale D. CRIES Scale

B Choice B is correct. Following the recent diagnosis of a chronic and incurable genetic condition such as cystic fibrosis, this family will require significant emotional support. Throughout the shift, the parents will likely have numerous questions regarding the need to follow up on genetic counseling, treatment options, prognosis, and/or resources, making Choice B the priority. Choice A is incorrect. Based on the information in this question, there is no indication that this infant's discharge is pending, nor has a pediatric pulmonology referral been written. Choice C is incorrect. Arranging for financial assistance is typically a lengthy, multi-step process. Procurement of financial assistance is the type of goal one would establish as a long-term goal for this family. Choice D is incorrect. The parents must understand the risks of conceiving additional children but at this time, arranging for parental genetic testing is not the priority.

You receive the change-of-shift report for an infant whose family has just been informed of the infant's cystic fibrosis diagnosis. As the nurse caring for this pediatric client and the family, which of the following should you prioritize? A. Arrange and schedule a follow-up appointment with a pediatric pulmonologist B. Provide emotional support for the family C. Arrange for financial assistance D. Arrange for parental genetic testing, as the parents mention they want another child soon

A Choice A is correct. You would suspect a grade 3 phlebitis and you would immediately stop the intravenous fluid. Grade 3 phlebitis is characterized by pain, a visible streak, site redness, and a palpable venous cord less than 1 inch. Grade 4 phlebitis is characterized by pain, a visible streak, site redness, a palpable venous cord more than 1 inch, and possible drainage. Lastly, as with all intravenous therapy, any suspicion of a complication is immediately addressed with the discontinuation of the intravenous line. Choice B is incorrect. The signs and symptoms in this question indicate the presence of phlebitis. However, it is not a grade 4 phlebitis. Additionally, after discontinuing the intravenous line, you would apply heat and not a cold compress onto the IV site. Choice C is incorrect. Although you would immediately stop the intravenous fluid infusion when an intravenous therapy complication occurs, this complication is not infiltration according to the signs and symptoms that are in the question. Choice D is incorrect. The signs and symptoms that are in the question do not indicate the presence of a catheter embolus; these signs and symptoms indicate the presence of another intravenous therapy complication. Additionally, when a catheter embolus occurs, a tourniquet would be placed proximal to the site to prevent migration and further damage.

Your client has continuous intravenous fluid replacement at 75 mL per hour. At 2 pm, the client complains about the intravenous line and states, "The IV is hurting me." You assess the site and note that it is red with a streak. You palpate the area and you can barely feel a venous cord. What would you suspect and what is the first thing that you would do? A. Grade 3 phlebitis: You would immediately stop the intravenous fluid infusion. B. Grade 4 phlebitis: You would immediately place a cool compress on the site. C. Infiltration: You would immediately stop the intravenous fluid infusion. D. Catheter embolus: You would immediately tourniquet the area distal to the site.


Kaugnay na mga set ng pag-aaral

The Crucible: Miscellaneous Questions

View Set

Chapter 8 - Understanding Power and Politics

View Set

Health Chapter 12 Lessons 1 and 2

View Set

Chapter 9: Skeletal muscle tissue and muscle organization

View Set

Guarantee Exam Questions Brian Caldwell

View Set

Introduction to Manufacturing - Exam 1

View Set

Fritz - Human Anatomy - 2.3 - 3.1 - Tissues, skin, skin diseases + disorders

View Set

Chapter 10: Life Span: Older Adults

View Set